*NURSING > EXAM REVIEW > I’ve given you all the different questions that PEDS HESI 2022 has had on the exam. All the answer (All)

I’ve given you all the different questions that PEDS HESI 2022 has had on the exam. All the answers are correct. good luck.

Document Content and Description Below

2022 PEDS HESI Test Bank- Over 1800 Q&A A+ Guide. (use CtrL + F to get to your specific Q&A) 7 y/o brought to school clinic because he fainted on the playground. 3 ft 7 in, 55 lbs, and BMI 20.9. W... hich assessment finding is most important to address? #Correct Answer- Since age 3 he has experienced exercise induced asthma The nurse is using the Ages and Stages Questionnaire (ASQ) to assess a 24-month-old. What is the best intervention for the nurse to initiate after the assessment is completed? • -A Review the child's birth history. • -B Provide the parents with a list of stimulating activities. • -C Meet with a social worker to review the results. • -D Assess for changes in the vital signs. #Correct Answer- • -B Provide the parents with a list of stimulating activities. 17 y/o with CF talks with school nurse about disease and wonders how it will effect getting married and having kids. Which relevant information would nurse include? #Correct Answer- • He is likely to have infertility problems and further evaluation 4 y/o taken to ER because of fever for last 24 hrs. No cough, pulling at ears, drooling, cyanotic, inspiratory stridor, expiratory wheeze. Which intervention is most important? #Correct Answer- Emergency trach Nurse giving IV antibiotics to a 16 month old with pneumonia. No allergies and been walking without assistance for 1 month. Which technique should select for administration? • A Administer the injection into the middle of the lateral aspect of the thigh. • B Use a needle length of ½ inch (1.25cm) to avoid deep tissue damage. •C Divide the gluteal area into quarters and give IM into the upper outer quadrant. •D Give in the arm, one to 2 inches (2.5 to 5.0 cm) below the acromion process. #Correct Answer- • A Administer the injection into the middle of the lateral aspect of the thigh. The nurse is examining an infant for possible cryptorchidism. Which exam technique should be used? A) Place the infant in side-lying position to facilitate the exam B) Hold the penis and retract the foreskin gently C) Cleanse the penis with an antiseptic-soaked pad D) Place the infant in a warm room and use a calm approach #Correct Answer- D) Place the infant in a warm room and use a calm approach An infant who has been diagnosed with a tracheoesophageal fistula (TEF). Which nursing intervention is indicated for this infant prior to surgical repair? A• Provide frequent sips of liquid B• Give isotonic enemas as prescribed C• Maintain nothing by mouth status D• Prepare the infant for a barium enema #Correct Answer- C• Maintain nothing by mouth status Nurse gets a call from a mother of a 10 y/o that just returned from camp with expanding circular red rash on arm. Mom asks what over the counter product is safe to use. How should the nurse respond? #Correct Answer- Explain need for immediate exam (lyme disease) What nutritional info to provide mom of a 6-month old regarding solid food? #Correct Answer- Initiate one at a time in 4-7 day intervals A child with hemophilia arrives at the clinic with a swollen knee after falling off a bicycle. What action should the nurse implement first? A• Initiate an IV site and begin infusing normal saline• Type and cross for possible transfusion B• Monitor the child's vital signs frequently C• Apply ice pack and compression dressing to knee #Correct Answer- C• Apply ice pack and compression dressing to knee A breast feeding mother returns to work when her infant is 5 months old. She is having difficulty pumping enough milk to meet her infant's dietary requirements. Which suggestion should the nurse provide to this mother? A• Mix infant formula with breast milk B• Supplement with an iron-rich formula C• Introduce baby food for one meal daily D• Offer a follow-up transitional formula #Correct Answer- B• Supplement with an iron-rich formula How should the nurse respond to the concerned parent of a 15-month-old who is not yet able to self-feed with a spoon? A• Tell parent to guide the child's hand using a spoon B• Suggest using foods that can be eaten with fingers C• Discuss possible causes for delay with self-feeding D• Encourage longer mealtimes to practice eating with a spoon #Correct Answer- B• Suggest using foods that can be eaten with fingers A toddler with hemophilia is being discharged from the hospital. Which teaching should the nurse include in the discharge instructions to the mother? A• Apply padding on the sharp corners of the furniture B• Prevent the child from running inside the house C• Give an 81 mg tablet of aspirin for pain relief D• Use a soft toothbrush for frequent cleaning #Correct Answer- A• Apply padding on the sharp corners of the furniture The healthcare provider prescribes antipyrine and benzocaine (Auralgan Otic), and anesthetic ear drop, for a two-year-old child with otitis media in the right ear. After positioning the child with the affect ear up, what action should the nurse take? A• Cleanse the ear canal with saline B• Put upward traction on the ear lobe C• Pull pinna of the ear down and back D• Gently massage in front of the ear #Correct Answer- C• Pull pinna of the ear down and back During a routine clinic visit, 5 y/o SBP >90th percentile. What action is implemented next? #Correct Answer- Measure two more times then take average Nurse caring for a 1 y/o with DM type 1. Mom asks how to know signs of hypoglycemia. Info to provide? #Correct Answer- Change in behavior and cold and clammy skin Change in behavior and cold and clammy skin #Correct Answer- 9% Nurse eval - child with atopic dermatitis. Which question should nurse ask while obtaining history? #Correct Answer- Has child displayed any symptoms of asthma or hay fever 1-month old vomiting forcefully times 3 days after meals. Afebrile, dehydrated and pyloric stenosis is suspected. What other findings will identify to confirm this? #Correct Answer- Olive shaped mass Hospitalized child stiffens and starts to seize as the nurse enters the room. What action should nurse take? #Correct Answer- Turn patient if possible, pad rails with available pillows and blankets, monitor time and progress 12 y/o boy with leukemia discharges with a WBC of 4000. He is scheduled to receive chemo as outpatient. What instruction should nurse include in discharge plan? #Correct Answer- Avoid buffet and salad bars Nurse is planning care for a newborn scheduled for cardiac catheter. Which occurrence poses greatest risk? #Correct Answer- Acute hemorrhage from access site after procedure When providing care for a kid in balanced suspension skeletal traction using them as splint and attach to right femur. Which intervention most important? #Correct Answer- Monitor pulses and sensation 10 y/o with IBS. Mom concerned she will experience developmental delays as a result. How should the nurse respond? #Correct Answer- Growth failure is a concern but not developmental delays Adolescent with AHL complaining of sores in mouth 2 days after start of chem. What activity should nurse implement? #Correct Answer- Frequent NS oral rinse and soft sponge toothbrush What snack is best to provide 6 y/o prescribed bed rest while receiving treatment for osteomyelitis? #Correct Answer- Milkshake During a well-child visit for their child, one of the parents who has an autosomal dominant disorder tells the nurse, "We don't plan on having any more children, since the next child is likely to inherit this disorder." How should the nurse respond? #Correct Answer- Confirm that there is a 50% chance of their future children inheriting the disorder. Nurse eval effects of thyroid therapy used to treat 5 months old with hypothyroidism. Which behavior indicates effective? #Correct Answer- Laughs readily, turns from back to side The HR for a 3 year old with CHF has steadily decreased over the last few hours, now is 76 bpm, the previous reading 4 hours ago was 110 bpm. Which additional finding should be reported immediately to a healthcare provider? #Correct Answer- BP 70/40 A 10-year-old boy has been seen frequently by the school nurse over the past three weeks after school begins in the fall. He complains of headaches, stomach aches, and difficulty sleeping. What intervention should the nurse implement? #Correct Answer- Ask the boy to describe a typical day at school A child with laryngotracheobronchitis (croup) received epinephrine 2 hours ago in the emergency room, and is now being prepared for discharge to home. The nurse should instruct the parents to take which action if the child's uncontrolled coughing reoccurs? #Correct Answer- Sit with the child in the bathroom with hot steam After receiving a single fluid bolus of 20 mL/kg of normal saline, a child's heart rate is 140 beats/minute, capillary refill is 6 seconds. The child is anxious and crying. Which intervention should the nurse implement? #Correct Answer- Repeat the normal saline bolus as prescribed A 3 year old girl who has been blind since birth is hospitalized because of as compound fracture of the femur and is now in traction. Which intervention is best for the nurse to implement to address this child's blindness? #Correct Answer- Request parents bring familiar objects such as a stuffed animal from home A child with possible Duchenne muscular dystrophy ( MD) undergoes an electromyogram (EMG). Following the procedure, the child's parents tell the nurse that the child is complaining of sore muscle. How should the nurse respond? A• Explain that muscle aches and pain are commonly experienced by children with this form of muscular dystrophy B• Advise the parents that children with chronic diseases may seek attention by reporting pain or other unpleasant symptoms C• Encourage the parents to motor the child's body temperature for the next 24 hours and report a rinse above 101 F D• Offer reassurance that muscle soreness following this procedures is temporary and does not indicate a problem #Correct Answer- D• Offer reassurance that muscle soreness following this procedures is temporary and does not indicate a problem 4-month old with inguinal hernia visible when crying. Does not hurt him. Parents ask is they should repair that now. Nurse's response based on what info? A• An inguinal hernia is treated as a surgical emergency B• Surgical repair is planned after successful toilet training C• An inguinal hernia is surgically repaired if persistent diarrhea occurs D• Surgical correction is indicated if the hernia is incarcerated #Correct Answer- D• Surgical correction is indicated if the hernia is incarcerated The nurse should instruct the parent of an 8-year-old child who has sickle anemia to alert for which complaint from the child? A• "I'm shorter than everyone else" B• "I'm really hot and thirsty" C• "I don't want to eat any vegetables" D• "I have to urinate every few hours" #Correct Answer- B• "I'm really hot and thirsty" The mother of 6-year-old girl is concerned about her child's obesity. The child's weight plots at the 75 percentile, and height at the 25 percentile. The child's body mass index (BMI) is at the 85 percentile for age and gender. Which interventions should the nurse implement? (Select all apply) A• Explain that the child is likely to grow into weight B• Determine the child's usual physical activity pattern C• Obtain the child's 3 - day diet history based on the mother's input D• Inquire as to whether or not the school has a physical education E• Tell the mother that girls hit their growth spurt before boys so eating #Correct Answer- • Determine the child's usual physical activity pattern • Obtain the child's 3 - day diet history based on the mother's input • Inquire as to whether or not the school has a physical education The nurse is assessing an infant with aortic stenosis and identifies bilateral fine crackles in both lung fields. Which additional finding should the nurse expect to obtain? A• Vigorous feeding and satiation B• Hemiplegia C• Fever D• Hypotension and tachycardia #Correct Answer- D• Hypotension and tachycardia Performing a routine exam on 6-month old at clinic. Records indicate baby weighed 3 kg at birth. Clinic uses pounds to describe weight. When assessing this child, approximately what weight in lbs should nurse consider to be within normal range? #Correct Answer- 12-15 lbs (6 lb x 2.2 kg) A 12-year-old is admitted to the hospital with possible encephalitis, and a lumbar puncture is schedule. Which information should the nurse provide concerning to this procedure? A• Explain that fluids cannot be taken for 8 hours before the procedure and 4 hours after the procedure B• Tell the child to expend loud clicking during the procedure that may be annoying C• Describe the side-lying, knee to the chest position that must be assumed during the procedure D• Reassure the child that there will be no restrictions on activity after the procedure is completed #Correct Answer- C• Describe the side-lying, knee to the chest position that must be assumed during the procedure Child vomiting for 3 days is admitted for fluid and electrolyte imbalance. What acid base imbalance is likely? #Correct Answer- Metabolic alkalosis A 3-month-old with myelomeningocele and atonic bladder is catheterized every 4 hours to prevent urinary retention. The home health nurse notes that the child has developed episodes of sneezing, urticaria, watery eyes, and a rash in the diaper area. What action is most important for the nurse to take? A) Auscultate the lungs for respiratory pneumonia B) Change to latex-free gloves when handling infant C) Draw blood to analyze for streptococcal infection D) Apply zinc oxide to perineum with each diaper change #Correct Answer- Teacher notifies school nurse that kid has nose bleed for no reason. What action should nurse implement first? #Correct Answer- Pinch nose and hold for 10 minutes The nurse has provided discharge teaching to the mother of a premature infant. Which statement by the mother would indicated that the understands the importance of making sure that her baby gets the monthly palivizumab (Synagis) injection? A• Palivizumab will help with neurological and physical development B• The medication will protect my baby from respiratory syncytial virus C• Palivizumab will prevent the development retinopathy of premature D• The monthly injections will baby's lung mature #Correct Answer- B• The medication will protect my baby from respiratory syncytial virus Which client requires immediate intervention by the nurse? A• A child with cystic fibrosis who is constipated B• A toddler with chicken pox who is scratching C• A child with acute renal failure and hyperkalemia D• An adolescent with a migraine and photophobia #Correct Answer- C• A child with acute renal failure and hyperkalemia Albumin 25% IV is prescribed for a child with nephrotic syndrome. Which assessment finding indicates to the nurse that the medication is having the desired effect? A) Weight gain B) Reduction of fever C) Improved caloric intake D) Reduction of edema #Correct Answer- D) Reduction of edema When developing a plan of care for adolescent with a recent diagnosis of DM type 1, nurse should instruct to eat a source of sugar if which symptom occurs? #Correct Answer- Profuse sweating A female of child - bearing age receives a rubella vaccination. She has two children at home, ages 13 months and 3 years. Which instruction is most important for the nurse to provide to this client? A• Tell the mother to isolate the children for 3 days B• Inquire if anyone in the family is allergic to eggs C• Encourage the client to immunize the children D• Assess family history for incidence of rubella #Correct Answer- C• Encourage the client to immunize the children The nurse administers digoxin (Lanoxin) to a 9-month-old infant with an apical heart rate of 160 beats per minute. Which apical pulse rate indicates that therapeutic effect of the medication has been achieved? A• 80 beats per minute B• 180 beats per minute C• 60 beats per minute D• 120 beats per minute #Correct Answer- D• 120 beats per minute During her sports physical examination, 15-year -old female requests oral contraceptives. She explains that she is sexually active and does not want her parents to know. What action should the nurse take? A• Explain that she needs parental approval to receive contraceptives B• Encourage the client to discuss her need for contraceptives with her parents C• Counsel the client about the risks and benefits of using oral contraceptives D• Tell the client how to receive a variety of oral contraceptives from the clinic #Correct Answer- C• Counsel the client about the risks and benefits of using oral contraceptives The nurse is assessing an 8-month-old who has a cough, axillary temperature of 100 F, and rhinorrhea. What information is most important for the nurse to obtain from this child's mother? A• Alcohol and drug intake of the mother B• Labor and delivery history of the infant C• Immunization status of the infant D• Living conditions #Correct Answer- C• Immunization status of the infant An 8 year-old child is admitted to the Emergency Department because of lower right quadrant pain, nausea, and vomiting. Which assessment of the abdomen should the nurse conduct after all other assessments are complete? A• Percussion B• Palpation C• Inspection C• Auscultation #Correct Answer- B• Palpation List the 4 defects associated with Tetralogy of Fallot. #Correct Answer- VSD, overriding aorta, pulmonary stenosis and right ventricular hypertrophy When does birth length double? #Correct Answer- by 4 years When does the child sit unsupported? #Correct Answer- 8 months When does a child achieve 50% of adult height? #Correct Answer- 2 years When does a child throw a ball overhand? #Correct Answer- 18 months When does a child speak 2-3 word sentences? #Correct Answer- 2 years When does a child use scissors? #Correct Answer- 4 years When does a child tie his/her shoes? #Correct Answer- 5 years List 2 contraindications for live virus immunization. #Correct Answer- Immunocompromised child or a child in a household with an immunocompromised individual. List 3 classic signs and symptoms of measles. #Correct Answer- 1. photophobia 2. confluent rash that begins on the face and spreads downward 3. Koplik spots on the buccal mucosa List the signs and symptoms of iron deficiency. #Correct Answer- Anemia, pale conjunctiva, pale skin color, atrophy of papillae on tongue, brittle/ridged/ spoon-shaped nails, and thyroid edema. List the laboratory findings that can be expected in a dehydrated child. #Correct Answer- Loss of bicarbonate/decreased serum pH, loss of sodium (hyponatremia), loss of potassium (hypokalemia), elevated Hct, and elevated BUN. How can the nurse BEST evaluate the adequacy of fluid replacement in children? #Correct Answer- Monitor urine output. What are the physical assessment findings for a child with asthma? #Correct Answer- Expiratory wheezing, rales, right cough, and signs of altered blood gases. What nutritional support should be provided for the child with cystic fibrosis? #Correct Answer- Pancreatic enzyme replacement, fat-soluble vitamins, and a high carbohydrate, high protein, moderate fat diet. Why is genetic counseling important for the cystic fibrosis family? #Correct Answer- The disease is autosomal recessive in its genetic pattern. Differentiate between a right to left and left to right shunt in cardiac disease. #Correct Answer- A left to right shunt moves oxygenated blood back through the pulmonary circulation. A right to left shunt bypasses the lungs and delivers unoxygenated blood to the systemic circulation causing cyanosis. Describe nursing interventions to reduce the workload of the heart. #Correct Answer- Small, frequent feedings or gavage feedings. Plan frequent rest periods. Maintain a neutral thermal environment. Organize activities to disturb child only as indicated. What position would best relieve the child experiencing a "tet" spell? #Correct Answer- Knee-chest position, or squatting. What are common signs of digoxin toxicity? #Correct Answer- Diarrhea, fatigue, weakness, nausea and vomiting. The nurse should check for bradycardia prior to administration. What cardiac complications are associated with rheumatic fever? #Correct Answer- Aortic valve stenosis and mitral valve stenosis. What medications are used to treat rheumatic fever? #Correct Answer- Penicillin, erythromycin, and aspirin. What are 2 nursing priorities for a newborn with myelomeningocele? #Correct Answer- Prevention of infection of the sac and monitoring for hydrocephalus (measure head circumference; check fontanel; assess neurological functioning). What teaching should parents of a newly shunted child receive? #Correct Answer- Signs of infection and increased ICP (decreased pulse, increased blood pressure). Shunt should not be pumped. Child will need revisions due to growth. Provide guidance for growth and development. State the 3 main goals in providing nursing care for a child experiencing a seizure. #Correct Answer- Maintain patent airway, protect from injury, and observe carefully. Describe the signs and symptoms of a child with meningitis? #Correct Answer- Fever, irritability, vomiting, neck stiffness, opisthotonos, positive Kernig's sign, positive Brudzinski's sign. Infant does not show all classic signs, but is very ill. What antibiotics are usually ordered for bacterial meningitis? #Correct Answer- Ampicillin, penicillin, and/or Chloramphenicol. What antecedent event occurs with acute glomerulonephritis? #Correct Answer- Beta-hemolytic strep infection What nursing actions are initiated for the newborn with suspected esophageal atresia with TEF? #Correct Answer- NPO immediately and suction secretions. Describe the pre-op nursing care for a child with Hirschsprung's disease. #Correct Answer- Check vital signs and take axillary temps. Provide bowel cleansing program and teach about colostomy. Observe for bowel perforation; measure abdominal girth. Describe what information families should be given when a child is receiving oral iron preparations. #Correct Answer- Give oral iron on an empty stomach and with vitamin C. Use straws to avoid discoloring teeth. Tarry stools are normal. Increase dietary sources of iron. Describe the sequence of events in a vasoocclusive crisis in sickle cell anemia. #Correct Answer- A vaso-occlusive crisis is caused by clumping of red blood cells which cannot get through the capillaries, causing pain and tissue/organ ischemia. Lowered oxygen tension affects the HgbS, which causes sickling of the cells. Nursing interventions and medical treatment for the child with leukemia are based on what 3 physiological problems? #Correct Answer- Anemia (decreased erythrocytes). Infection (neutropenia). Bleeding thrombocytopenia (decreased platelets). What are the signs and symptoms of compartment syndrome? #Correct Answer- Abnormal neurovascular assessment: cold extremity, severe pain, inability to move the extremity, and poor capillary refill. How is skeletal traction applied? #Correct Answer- Skeletal traction is maintained by pins or wires applied to the distal fragment of the fracture. What are the signs and symptoms of congenital dislocated hip in infants? #Correct Answer- Unequal skin folds of the buttocks, ortalani sign, limited abduction of the affected hip, and unequal leg lengths. What instructions should the child with scoliosis receive about the Milwaukee brace? #Correct Answer- Wear the brace 23 hours per day. Wear t-shirt under brace. Check skin for irritation. Perform back and abdominal exercises. Modify clothing. Encourage the child to maintain normal activities as able. The nurse is caring for a 3-year old child who is 2 hours postop from a cardiac catheterization via the right femoral artery. Which assessment finding is an indication of arterial obstruction? A. Blood pressure trend is downward and pulse is rapid and irregular. B. Right foot is cool to the touch and appears pale and blanched. C. Pulse distal to the femoral artery is weaker on the left foot than right foot. D. The pressure dressing at right femoral area is moist and oozing blood. #Correct Answer- B. Right foot is cool to the touch and appears pale and blanched. Following a motor vehicle collision, a 3-year old girl has a spica cast applied. Which toy is best for the nurse for this 3 year old child? A. Duck that squeaks. B. Fashion doll and clothes. C. Set of cloth and hand puppets. D. Hand held video game. #Correct Answer- C. Set of cloth and hand puppets. An infant with tetralogy of Fallot becomes acutely cyanotic and hyperpneic. Which action should the nurse implement first? A. Administer morphine sulphate. B. Start IV fluids. C. Place the infant in a knee-chest position. D. Provide 100% oxygen by face mask. #Correct Answer- C. Place the infant in a knee-chest position. A child admitted with diabetic ketoacidosis is demonstrating Kussmaul respirations. The nurse determines that the increased respiratory rate is a compensatory mechanism for which acid base alteration? A. Metabolic alkalosis. B. Respiratory acidosis. C. Respiratory alkalosis. D. Metabolic acidosis. #Correct Answer- D. Metabolic acidosis. 7 years old is admitted to the hospital with persistent vomiting, and a nasogastric tube attached to low intermittent suction is applied. Which finding is most important for the nurse to report to the healthcare provider? A. Gastric output of 100 mL in the last 8 hours. B. Shift intake of 640 mL IV fluids plus 30 mL PO ice chips. C. Serum potassium of 3.0 mg/dL. D. Serum pH of 7.45 #Correct Answer- C. Serum potassium of 3.0 mg/dL. The nurse is evaluating diet teaching for a client who has nontropical sprue (celiac disease). Choosing which food indicates that the teaching has been effective? A. Creamed corn B. Pancakes. C. Rye crackers. D. Cooked oatmeal. #Correct Answer- A. Creamed corn During a well-baby check, the nurse hides a block under the baby's blanket, and the baby looks for the block. Which normal growth and development milestone is the baby developing? A. Separation anxiety. B. Associative play. C. Object prehension. D. Object permanence. #Correct Answer- D. Object permanence. The nurse is measuring the frontal occipital circumference (FOC) of a 3-months old infant, and notes that the FOC has increased 5 inches since birth and the child's head appears large in relation to body size. Which action is most important for the nurse to take next? A. Measure the infant's head-to-toe length. B. Palpate the anterior fontanel for tension and bulging. C. Observe the infant for sunken eyes. D. Plot the measurement on the infant's growth chart. #Correct Answer- B. Palpate the anterior fontanel for tension and bulging. The nurse is preparing a 10 year old with a lacerated forehead for suturing. Both parents and 12 year old sibling are at the child's bedside. Which instruction best supports family? A. While waiting for the healthcare provider, only one visitor may stay with the child. B. All of you should leave while the healthcare provider sutures the child's forehead. C. It is best if the sibling goes to the waiting room until the suturing is completed. D. Please decide who will stay when the healthcare provider begins suturing. #Correct Answer- D. Please decide who will stay when the healthcare provider begins suturing. The nurse is planning for a 5-month old with gastroesophageal reflux disease whose weight has decreased by 3 ounces since the last clinic visit one month ago. To increase caloric intake and decrease vomiting, what instructions should the nurse provide this mother? A. Give small amounts of baby food with each feeding. B. Thicken formula with cereal for each feeding. C. Dilute the childs formula with equal parts of water. D. Offer 10 % dextrose in water between most feedings. #Correct Answer- B. Thicken formula with cereal for each feeding. While teaching a parenting class to new parents the nurse describes the needs of infants and toddlers regarding discipline and limit setting. What is the most important reason for implementing such parenting behaviors? A. Children need help in developing social skills. B. This age child fears loss of self control. C. They provide the child with a sense of security. D. Children must to learn to deal with authority. #Correct Answer- C. They provide the child with a sense of security. The parents of a newborn infant with hypospadia are concerned about when the surgical correction should occur. What information should the nurse provide? A. Repair should be done by one month to prevent bladder infection. B. To form a proper urethra repair, it should be done after sexual maturity. C. Repairs typically should be done before the child is potty trained. D. Delaying the repair until school age reduces castration fears. #Correct Answer- C. Repairs typically should be done before the child is potty trained. Which drink choice on a hot day indicates to the nurse that a teenager with sickle cell anemia understands dietary consideration related to the disease? A. Milkshake. B. Iced tea. C. Diet cola. D. Lemonade. #Correct Answer- D. Lemonade. The nurse is assessing an infant with diarrhea and lethargy. Which finding should the nurse identify that is consistent with early dehydration? A. Tachycardia. B. Bradycardia. C. Dry mucous membrane. D. Increased skin turgor. #Correct Answer- A. Tachycardia. While auscultating the lung sounds of a 5 year old Chinese boy who recently completed antibiotic therapy for pneumonia, the nurse notices symmetrical, round, bruise-like blemishes on his chest. What action is best for the nurse to take? A. Identify the antibiotic used to treat the pneumonia. B. Inquire about the use of alternative methods of treatment. C. Ask the parents if the child has been in a recent accident. D. Report suspected child abuse to the authorities. #Correct Answer- B. Inquire about the use of alternative methods of treatment. A child with acute lymphocytic leukemia (ALL) who is receiving chemotherapy via a subclavian IV infusion, has an oral temperature of 103 degrees. In assessing the IV site, the nurse determines that there are no signs of infection at the site. Which intervention is the most important for the nurse to implement? A. Obtain specimen for blood cultures. B. Assess the CBC. C. Monitor the oral temperature every hour. D. Administer acetaminophen as prescribed. #Correct Answer- A. Obtain specimen for blood cultures. A child who weights 25 kg is receiving IV ampicillin 300 mg/kg/24 hours in equally divided doses every 4 hours. How many mg should the nurse administer to the child for each dose? #Correct Answer- 1875mg The nurse is caring for an infant scheduled for reduction of intussusceptions. The day before the scheduled procedure the infant passes a soft-formed brown stool. Which intervention should the nurse implement? A. Instruct the parents that the infant needs to be NPO. B. Notify the healthcare provider of the passage of brown stool. C. Obtain a stool specimen for laboratory analysis. D. Ask the parents about recent changes in the infant's diet. #Correct Answer- B. Notify the healthcare provider of the passage of brown stool. The mother of a 4-month old asks the nurse for advice in preventing diaper rash. What suggestion should the nurse provide? A. At diaper change generously powder the baby's diaper area with talcum powder to promote dryness. B. Wash the diaper area every 2 hours with soap and water to help prevent skin breakdown. C. Use a barrier cream, such as zinc oxide, which does not have to be completely removed with each diaper change. D. Place a cloth diaper inside the disposable diaper for overnight periods when increased wearing time is likely. #Correct Answer- C. Use a barrier cream, such as zinc oxide, which does not have to be completely removed with each diaper change. Which statement by a school aged client going to summer camp indicates the best understanding of the mode of transmission of Lyme disease? A. I'll cover my mouth with a wet cloth if there's too much dust blowing. B. Cuts and scrapes need to be washed out and covered right away. C. I'm not going to swim where the water is standing still or feels too hot. D. I have to wear long sleeves and pants when we're hiking around the pond. #Correct Answer- D. I have to wear long sleeves and pants when we're hiking around the pond. The nurse is evaluating the effects of thyroid therapy used to treat a 5 months old with hypothyroidism. Which behavior indicates that the treatment has been effective? A. Laughs readily, turns from back to side. B. Has strong Moro and tonic neck reflexes. C. Keeps fists clenched, opens hands when grasping an object. D. Can lift head, but not chest when lying on abdomen. #Correct Answer- A. Laughs readily, turns from back to side. The HR for a 3 year old with a congenital heart defect has steadily decreased over the last few hours, now it's 76 bpm, the previous reading 4 hours ago was 110 bpm. Which additional finding should be reported immediately to a healthcare provider? A. Oxygen saturation 94%. B. RR of 25 breaths/minute. C. Urine output 20 mL/hr. D. BP 70/40. #Correct Answer- D. BP 70/40. 2 year old is admitted to the hospital with possible encephalitis, and a lumbar puncture is scheduled. Which information should the nurse provide this child concerning the procedure? A. Describe the side-lying, knees to chest position that must be assumed during the procedure. B. Tell the child to expect loud clicking noises during the procedure that may be slightly annoying. C. Reassure the child that there will be no restrictions on activity after the procedure is completed. D. Explain that fluids cannot be taken for 8 hours before the procedure and for 4 hours after the procedure. #Correct Answer- A. Describe the side-lying, knees to chest position that must be assumed during the procedure. the parenst of a 3 y/o boy who has Duchenne muscular dystrophy (DMD) ask "how can our son have this disease? We are wondering if we should have any more children" What information should the nurse provide these parents? A. This is an inherited X-linked recessive disorder, which primarly affects male children in the family B. The male infant had a viral infectrion that went unnoticed and iuntreated, so mucle damage was incurred C. The XXXX muscle groups of males can be impacted by a lack of the protein dystrophyn in the mother D. Birth trauma with a breech vaginal birth causes damage to the spinal cord, thus weakening the muscles #Correct Answer- A. This is an inherited X-linked recessive disorder, which primarly affects male children in the family The nurse finds a 6 month old infant unresponsive and calls for help. After opening the airway and finding the XXXX the infant is still no breathing. Which action should the nurse take? A. Palpate femoral pulse and check for regularity B. Deliver cycles of 30 chest compressions and 2 breaths C. Give two breath that makes the chest rise D. Feel the carotid pulse and check for adequate breathing #Correct Answer- C. Give two breath that makes the chest rise A 3 year old with HIV infection is staying with a foster family who is caring for 3 other foster children in their home. When one of the children acquires pertussis, the foster mother calls the clinic and asks the nurse what she should do. Which action should the nurse take first? A. Remove the child who has HIV from the foster home B. Report the exposure of the child with HIV to the health department C. Place the chuld who has HIV in reverse isolation D. Review the immunization documentation of the child who has HIV #Correct Answer- D. Review the immunization documentation of the child who has HIV A 16 y/o female student with a history of asthma controlled with both an oral antihistamine and an albuterol (Provenfil) metere-dose inhaler (MDI) comes to the school nurse. The student complains that she cannot sleep at night, feels shaky and her heart feels like it is "beating a mile per minute" Which information is most important for the nurse to obtain? a. When she last took the antihistamine b. When her last Asthma attack occurred c. Duration of most asthmas attacks d. How often the MDI is used daily #Correct Answer- d. How often the MDI is used daily The nurse is assessing a child for neurological soft signs, which finding is most likely demonstrated in the child's behavior? a. Inability to move tongue in a direction b. Presence of vertigo c. Poor coordination and sense of position d. Loss of visual acuity #Correct Answer- c. Poor coordination and sense of position The nurse is assessing an infant with pyloric stenosis. Which pathophysiological mechanism is the most likely consequence of this infant's clinical picture? a. Metabolic alkalosis b. Respiratory acidosis c. Metabolic acidosis d. Respiratory Alkalosis #Correct Answer- a. Metabolic alkalosis A 4 month-old girl is brought to the clinic by her mother because she has had a cold for 2 o 3 days and woke up this morning with a hacking cough and difficulty breathing. Which additional assessment finding should alert the nurse that the child is in acute respiratory distress? a. Bilateral bronchial breath sounds b. Diaphragmatic respiration c. A resting respiratory rate of 35 breathe per minute d. flaring of the nares #Correct Answer- d. flaring of the nares a two year old boy begins to cry when the mother starts to leave. What is the nurse's best response in this situation? a. Let me read this book to you b. Two years old usually stop crying the minute the parent leaves c. Now be a big boy. Mommy will be back soon d. Let's wave bye-bye to mommy #Correct Answer- a. Let me read this book to you A two year old child with a heart failure (HF) is admitted for replacement of a graft for coarctation of the aorta. Prior to administering the next dose of digoxin (Lanoxin) the nurse obtains an apical heart rate of 128 bpm. What action should the nurse implement? a. Determine the pulse deficit b. Administer the scheduled dose c. calculate the safe dose range d. review the serum digoxin level #Correct Answer- b. Administer the scheduled dose A child with leukemia is admitted for Chemotherapy and the nursing diagnosis "altered nutrition, less those body requirements related to anorexia, nausea and vomiting" is identified. Which intervention the nurse included in this child plan of care? a. Encourage a variety of large portions of food at every meal b. Allow the child to eat any food desired and tolerated c. Recommended eating the food as sibling eat at home d. Restrict food brought form fast food restaurants #Correct Answer- b. Allow the child to eat any food desired and tolerated a 6 year old who has asthma is demonstrating a prolonged expiratory phase and wheezing and has a35% of personal best peak expiratory flow rate (PEFR) based on these finding, actions should the nurse take first? a. Administer a prescribed bronchodilator b. Encourage the child to cough and deep breath c. Report findings to the heath care provider d. determine what triggers precipitated this attack #Correct Answer- a. Administer a prescribed bronchodilator The nurse plans to administer 10 mcg/kg of digoxin elixir as a loading dose to a child who weights 55 pounds. Digoxin is available as elixir of 50 mcg/ml. How many ml of the digoxin elixir should the nurse administer to this child? #Correct Answer- 5 mL the nurse observes a mother giving her 11 month-old ferrous sulfate, followed by two onces of orange juice. What should the nurse do next? a. suggest placing the iron drops in the orange juice and feed the infant b. Tell the mother to follow the iron drops with formula instead of orange juice c. instruct the mother to feed the infant nothing in the next 30 minutes after the iron d. Give positive feedback about the way she administered the sulfate #Correct Answer- d. Give positive feedback about the way she administered the sulfate Which nursing intervention is most important to include in the plan of care for a child with acute glomerulonephritis A. encourage fluid intake b. promote complete bed rest c. weight the child daily d. administer vitamin supplements #Correct Answer- c. weight the child daily During a well baby visit the parents explain that a soft bulge appears in the groin of their 4 month old son when he cries or strain stooling. The infant is schedule for surgical repair of the inguinal; hernia in two weeks. The parent should be instructed to take which measure if the hernia becomes incarcerated prior to the surgery? a. Use rectal thermometer for straining on stool b. Gently manipulate the hernia for reduction c. Offer oral electrolyte fluids for comfort d. Give acetaminophen or aspirin for crying #Correct Answer- b. Gently manipulate the hernia for reduction A 16 year old male client who has been treated in the past for a seizure disorder is admitted to the hospital. Immediately after admission he begins to have a grand mal seizure. Which action should the nurse take? a. Obtain assistance in holding him to prevent injury b. Observe him carefully c. Call a CODE d. Place a padded tongue blade between the teeth #Correct Answer- b. Observe him carefully The mother of a 9 month old who was diagnosed with respiratory syncytial virus yesterday calls the clinic to inquire if it will be all right to take her infant to the first b-day party of a friend's child the following day. What response should the nurse provide this mother? a. The child will not longer be contagious, no need to take any further precaution b. Make sure there are not children under the age of 6 months around the infected child c. The child can be around other children but should wear mask at all times d. Do not expose other children to RSV. It is very contagious even without direct contact #Correct Answer- d. Do not expose other children to RSV. It is very contagious even without direct contact When screening a 5 year old for strabism, what action should the nurse take A. Have the child identify colored patterns on polychromatic cards B. Direct the child through the six cardinal position of glaze C. Inspect the child for the setting sun sign D. Observe the child for blank, sunken eyes #Correct Answer- B. Direct the child through the six cardinal position of glaze The nurse is assessing a 6 month old infant. Which response requires further evaluation by the nurse? A. Has doubled birth weight B. Turn head to locate sound C. Plays pick a boo D. Demonstrate startle reflex #Correct Answer- D. Demonstrate startle reflex A child is brought to the clinic complaining of fever and joins pain, and is DX with rheumatic fever. When planning care for this child what is the goal of nursing care? A. Reduce fever B. Maintain fluid and electrolytes C. Prevent cardiac damage D. Maintain join mobility and function #Correct Answer- C. Prevent cardiac damage The nurse working on the pediatric unit takes two 8-year old girls to the playroom. Which activity is best for the nurse to plan for these girls? A. Selecting a board game B. Playing Doctor and nurse C. Watching cartoon on TV D. Coloring, cutting and pasting #Correct Answer- A. Selecting a board game The nurse is developing the plan of care for a hospitalized child with von Willebrand disease. What priority nursing intervention should be included in this child plan of care A. Reduce exposure to infection B. Eliminate contact with cold grafts (crafts? Is not legible) C. Guard against bleeding injuries D. Reduce contact with other children #Correct Answer- C. Guard against bleeding injuries How should the nurse instruct the parents of a 4 month old with seborrheic dermatitis (cradle cap) to shampoo the child's hair? A. Use a soft brush and gently scrub the area B. B. Avoid scrubbing the scalp until the scales disappear C. Avoid washing the child's hair more than once a week D. Use soap and water and avoid shampoos #Correct Answer- A. Use a soft brush and gently scrub the area Prior to discharge, the parents of a child with cystic fibrosis are demonstrating chest physiotherapy (CPT) that they will perform for their child at home. Which action requires intervention by the nurse? A. Plan to perform CPT when the child awakens in the morning B. A Copped hand is used when percussing the lung field C. A bronchodilator is administered before starting CPT D. The child is placed in a supine position to begin percussion #Correct Answer- D. The child is placed in a supine position to begin percussion When assessing the breath sounds of an 18 month old child who is crying, what action should the nurse take? A. Document that the assessment is not available because the child is crying B. Ask the parents to quiet the child so breath sounds can be auscultated C. Allow the child to initially play with stethoscope, and distract during auscultation D. Auscultate and document breath sounds, noting that the child was crying at the time #Correct Answer- C. Allow the child to initially play with stethoscope, and distract during auscultation The mother of a one month old calls the clinic to report that the back of her infant is flat. How should the nurse respond A. Turn the infant on the left side braced against the crib when sleeping B. Prop the infant in a sitting position with a cushion when no sleeping C. Place a small pillow under the infant's head while lying on the back D. Position the infant on the stomach occasionally when awake and active #Correct Answer- D. Position the infant on the stomach occasionally when awake and active Which nursing intervention is most important to assist in detecting hypopituitarism and hyperpituitarism in children A. Carefully recording the height and weight of children to detect inappropriate growth B. Performing head circumference measurements on infants under one year of age C. Assessing for behavioral problems at home and school by interviewing the parents D. Noting a tracked weight gain without a gain in height on a growth chart #Correct Answer- A. Carefully recording the height and weight of children to detect inappropriate growth A 7 year old child is admitted to the hospital with acute glomerulonephritis (AGN). When obtaining the nursing history which finding should the nurse expect to obtain? A. High blood cholesterol level on routine screening B. Increased thirst and urination C. A recent strep throat infection D. A recent DPT immunization #Correct Answer- C. A recent strep throat infection The nurse plans to screen only the highest risk children for scoliosis. Which group of children should the nurse screen first A. Girls between ages 10 and 14 B. Boys between ages 10 and 14 C. Boys and girls between 12 and 14 D. Boys and girls between 8 and 12 #Correct Answer- A. Girls between ages 10 and 14 In assessing a 10 year old newly diagnosed with osteomyelitis, which information is most for the nurse to obtain A. Recent recurrence of infections B. Cultural heritage and belief C. Family history of bone disorder D. Occurrence of increased fluid intake #Correct Answer- A. Recent recurrence of infections A 3 year old boy in a daycare facility scratches his head frequently and the nurse confirms the presence if head lice. The nurse washes the child's hair with permethrin (Nix) shampoo and call his parents. What instructions should the nurse provide to the parents about treatment of head lice? A. Wash the child's bed linens and clothing In hot soapy water B. Dispose of the child's brushes, comb's and other hair accessories C. Rewash the child's hair following a 24 hour isolation period D. Take the child to a hair salon for a shampoo and shorter haircut #Correct Answer- A. Wash the child's bed linens and clothing In hot soapy water The nurse on a pediatric unit observes a distraught mother in the hallway scolding her 3 year old son for wetting his pants. What initial action should the nurse take? A. Suggest that the mother consult a pediatric nephrologists B. Provide disposable training pants while calming the mother C. Refer the mother to a community parent education program D. Inform the mother that toilet training is slower for boys #Correct Answer- D. Inform the mother that toilet training is slower for boys A 4-month old boy inguinal hernia that is visible when he cries, but it does not cause him discomfort. His parents ask if the hernia should be repaired now. The Nurse's response should be based on what information. A. An inguinal hernia is treated as a surgical emergency B. Surgical repair is planned after successful toilet training. C. An inguinal hernia is surgically repaired if persistent diarrhea occurs. D. Surgical correction is indicated if the inguinal hernia is incarcerated. #Correct Answer- D A hospitalized child stiffens and starts to seize as the nurse enters the room.What actions should the nurse take? (select all that apply) A. Turn client to the side if possible B.Pad aside rails with available pillows an blankets . C.Instruct the parents to leave the room. D.Notify the emergency response team. E.Monitor duration an progress of the seizure. #Correct Answer- A, D, E A 16-year old adolescent with acute myclocytic leukemia receiving chemotherapy.(CT) Via an implanted medication port at the out-patient oncology clinic .What action should the nurse implement when the infusion is complete? A. initiate an infusion of normal saline B. Administer Zofran C. Flush the metiport with saline and heporin solution D.Obtain blood samples for RBC, WBC, Platelets #Correct Answer- C The nurse is planing care for a 5-month old with gastroesophagel reflux disease whose weight has decreased 3 ounces the last clinic visit one moth ago. To increase caloric intake and decrease vomiting, what instruction should the nurse provide this mother? A. Dilute the child's formula with equal part of water B Offer 10% dextrose in water between mot feedings C Give small amounts of baby food with each feeding D Thicken formula with the cereal for each feeding #Correct Answer- C In caring for an client with acute epiglottitis, ,which nursing action takes priority? A obtain a stat CBC B Prepare for endotracheal intubation C Auscultate breath sounds D Apply ice packs to the neck #Correct Answer- A A 4-Year old boy was recently diagnosed with Duchenne muscular dystrophy (DMD). Which characteristic of the disease is most important for the nurse to focus on during the initial teaching? A Muscular strength can be regained with physical exercise and therapy. B Growth and development have been abnormal since birth C Respiratory dysfunction and aspiration are prime concerns at this stage of the disease. D Lower legs become progressively weaker, causing a wadding, unsteady gait. #Correct Answer- D Which client requires immediate intervention by the nurse? A A toddler with chicken pox who is scratching B An adolescent with a migraine and photophobia. C A child with cystic fibrosis who is constipated D A child with acute renal failure an hyperkalemia #Correct Answer- D The nurse is using the Ages and Stages Questionnaire (ASQ) to asses a 24-month old child.What is the best intervention for the nurse to initiate after the assessment is completed? A Assess for changes in the vital sign B Review the child's birth history C Provide the parents with a list of stimulating activities. D Meet with a social worker to review results. #Correct Answer- C The nurse is planning care for a newborn infant scheduled for a cardiac catheterization. Which occurrence poses the greatest risk for this child? A Loss of pulses proximal to the entry site of the cardiac catheter B Allergic response to the plastics in the catheter used for catheterization. C Acute hermorrhage from the entry site of the catheter after the procedure D Fever associated with nausea and vomiting after the procedure. #Correct Answer- C The school nurse is presenting a seminar to parents about child safety that ficus on prevention of spinal cord injuries. What information is most important for the nurse to include in the teaching plan? A Trampoline activities of school- aged children shouldn't be supervised by adults B Protective gear to prevent neck flexion should be worn during contact sports. C Seat belt and car seats laws for use in motor vehicles should be reinforced. D Monkey bars should be removed from school playgrounds to reduce falls. #Correct Answer- C A mother brings her 2-year-old son to the clinic because he has been crying and pulling on his earlobe for the past 12 hours. The child's oral temperature is 101.2 degrees Fahrenheit(38.4 degrees Celsius) which intervention should the nurse implement? a Provide parent education to prevent recurrence. B Cleanse purulent exudate from the affected ear canal. C Apply a tropical antibiotic to the periauricular area D Ask the mother if the child has had a runny nose #Correct Answer- D The parents of a 4-week-old infant phone the pediatric clinic to report that their infant eats well but vomits after feeding. To differentiate between normal regurgitation ans pyloric stenosis, which information is most important for the nurse to obtain? A Level of infant's distress after vomiting. B Degree of forcefulness of vomiting episodes C Odor and texture associated with emesis D Position of the infant when vomiting occur #Correct Answer- B A female of a child-bearing age receives a rubella vaccination. She has two children at home, ages 13 moths and 3 years. Which instruction is most important for the nurse to provide to this client? A Tell the mother to isolate the children for 3 days B Inquire if anyone in the family is allergic to eggs C Encourage the client to immunize the children D Asses family history for incidence of rubella #Correct Answer- C A mother is concerned that her 3-year-old son wants to play with female doll figures. The child is not interested in building blocks,truck,or other typical "Boy" toys. How should the nurse respond to the mother's concern? A Letting male toddlers play with female-typed toys can have negative effect B Replacing female doll figures with male doll figures reinforces masculinity. C Exploring different roles in imaginary play is typical wt this age D Experimenting with different toys is an acceptable behavior #Correct Answer- D A 2-year-old child with heart failure (HF) is admitted for replacement of a graft for coordination of the aorta. Prior to administering the next dose of digoxin ( Lanoxin), the nurse obtains an apical heart rate of 128 beats/ minute. What action should the nurse implement? A Determine the pulse deficit B Calculate the safe dose range C Administer the scheduled dose D Review the serum digoxin level #Correct Answer- D Which nursing intervention is most important to assist in detecting hypopituitarism and Hyperpituitarism in children? A Nothing a marked weight gain without a gain in height on a growth chart B Performing head circumference measurements on infants under one year of age. C Assessing for behavioral problems at home and school by interviewing the parents. D Carefully recording the height and weight of children to detect inappropriate growth rates #Correct Answer- D The nurse is assiting the mother of a child with phenylketonuria (PKU) to select foods that are keeping with the child'd dietary restrictions. Which foods are contraindicated for this child? A High fat foods B Foods sweetened with aspartame C Wheat products D High calorie foods #Correct Answer- B During a routine physical exam, a male adolescent client tell the nurse, "Sometimes,my mother gets angry because I want to be with my own friends." What is the best initial response by the nurse? A Offer to discuss his concerns together with his mother B Ask about the client's response to his mother's anger C Determine if his friends are engaged in unsafe behavior D Offer reassurance that his mother's concern is normal #Correct Answer- B Which response demonstrates that the mother of a young girl with a urinary tract infection (UTI)Understands home care for the child? A I will give the antibiotics until she does not complain of burning anymore. B I will bring her back to the doctor's office for another urine test. C I will make sure she wipes from back to front after she uses the bathroom D I will refill the prescription for antibiotics if her symptoms are still present after taking these #Correct Answer- D The nurse is administering an oral medication to a reluctant preschool-aged boy. Which intervention should the nurse implement? A Advise the parents that they need to give the medication B Use a straightforward approach with the child C Mix the medication in with the child's favorite breakfast cereal D Offer to bring the medicine back later in the day #Correct Answer- D When caring for a child with sickles cell disease,The nurse knows that the child will most likely exhibit which sign when experiencing a sickle cell crisis? A Decreased hemoglobin B Pain C infection D Dehydration #Correct Answer- B The nurse is caring for a one-year-old boy who has type 1 diabetes mellitus (DM). His mother asks how will she recognize hypoglycemia in her infant who cannot tell her how he feels. Which information should the nurse provide. A The baby's breath smells sweet when the sugar and blood ketone levels are high B Hypoglycemia in infants causes changes in behavior and cold clammy C Weight loss and a good appetite often occur when a baby's glucose levels change D Excess urination and dry skin are common indicators of hypoglycemia #Correct Answer- B An 8-year-Old child is Admitted to the emergency department because of lower right quadrant pain,nausea, and vomiting. Which assessment of the abdomen should the nurse conduct after all other assessments are complete? A Percussion B Palpation C Inspection D Auscultation #Correct Answer- B When administering indomethacin (Indocrin) to a premature infant who has patent ductus arteriosus, the nurse should anticipate with outcome? A Decreased cardiac murmur B Increased number of red blood cells C Decreased urinary output D increased respiratory effort #Correct Answer- B A hospitalized child stiffens and starts to seize as the nurse enters the room. What actions should the nurse take? ( Select all the apply) A Turn client to the side if possible B Pad aside rails with available pillows and blankets. C Instruct the parents to leave the room D Notify the emergency response team E Monitor duration and progress of the seizure #Correct Answer- A, b, e A middle school male student was recently diagnosed with attention-deficit Hyper Disorder (ADHD) and is having trouble withe his grades. He is referred to the school nurse by the teacher because he continues to have learning problems. which action should the school nurse take? A Refer the child to the school counselor for educational testing. B Seek the advice of the school principal regarding the child's learning needs C Ask the parents to become involved in helping the child with his homework D Ask the parents to have the child seen by a clinical psychologist #Correct Answer- D A 10 year old boy has been seen frequently by school nurse over the past three weeks after school begins in the fall. He reports headaches, stomach aches, and difficulty sleeping. What intervention should the nurse implement? A Conduct a complete neurological assessment B Ask the boy to descri)be a typical day at school C Counsel the parent to pay more attention to the child D Compare the child vital signs over the past three weeks #Correct Answer- B A 6 year old child is diagnosed with rheumatic fever and demonstrates associated chorea(Sudden aimless movements of the arms an leg) Which information should the nurse provide to the parent? A Permanent life style changes need to be able to promote safety in the home B Consistent discipline is needed to help the child control the movements C Muscle tension is decreased with fine motor skill projects, so these activities should be encouraged D The chorea or movement are temporary and will eventually disappear #Correct Answer- D A 3 year old girl who has been blind since birth is hospitalized because of a compound fracture of the femur and is now in traction. which intervention is best for the nurse to implement to address this child's blindness? A Play a game where the child must identify unfamiliar sounds in the environment B use a touch tour to allow the child to familiarize her self with the room layout C Request parents to bring in familiar objects such as a stuffed animal from home D Person the child's self car activities until the child is not longer in traction #Correct Answer- C The nurse has provided discharge teaching to the mother of a premature infant. Which statement by the mother would indicate that she understands the importance of making sure that her baby get the monthly palivizumab ( Synagis) injection? A Palivizumb will help with neurological and physical development B The medication will protect my baby from respiratory C Palivizumab will prevent the development of the retinopathy of prematurity D The monthly injections will help my baby's lungs mature. #Correct Answer- B A toddler with hemophilia is being discharged from the hospital. Which teaching should the nurse include in the discharge instructions to the toddler? A Apply padding on the sharp corners or the furniture B Prevent the child from running inside the house C Give an 81 mg tablet of aspirin for pain relief D Use a soft bristle toothbrush for frequent cleaning #Correct Answer- The teacher notifies the school nurse that a child's nose is bleeding for no reason. What action should a nurse implement first? A Tip child's head back to avoid swallowing B Pinch the nose using thumb and finger for 10 minutes C Insert a sterile cotton ball in the nares that it is bleeding D Apply an ice compress to the child's nose right away. #Correct Answer- The nurse is conducting an admission assessment of an 11- month old infant with congestive heart failure who is scheduled for repair of restenosis of coaractaion of the aorta that was repaired 4 day after birth. Findings include blood pressure higher in arms than lower extremities, pounding brachial pulses, and slightly palpable femoral pulses. What pathophysiology mechanism supports these findings. A The aortic semilunar valve obstructs blood flow into systemic circulation B An opening in the atrial septum causes a murmur due to turbulent left to right shunt C The lumen of the aorta reduces the volume of blood flow to the lower extremities D The Pulmonic valve prevents adequate blood volume into the pulmonary circulation #Correct Answer- A school age child is diagnosed with hypothyroidism. Which of the following should the nurse include when planning care for the client? A Provide thyroid supplement medication as prescribed for a heart rate of greater than 100 beats per minute B Restrict fresh fruits and vegetables in the diet C Instruct the parents and client regarding the need for life-long supplemental therapy D Explain that the child's maximum growth will not be achieved. #Correct Answer- Refer child to the healthcare provider and schedule evaluation of blood pressure in two weeks. #Correct Answer- The nurse is conducting an admission assessment of an 11-months old infant with congestive heart failure who is scheduled repair of restenosis of coarctation of the aorta that was repaired 4 days after birth. Findings include blood pressure higher in the arms than the lower, with pathophysiologic mechanism support these findings? #Correct Answer- The lumen of the aorta reduces the volume of flow to the lower extremities A child is to receive vancomycin (Vancocin) 20 mg/kg IV one hour before a scheduled procedure. The child weight 77 pounds. How many mg of the medication should the nurse prepare to administer? (Enter numeric value only) #Correct Answer- 700mg A toddler with hemophilia is being discharged from the hospital. Which teaching should the nurse include in the discharge instructions to the mother? #Correct Answer- Apply padding on the sharp corners of the furniture The nurse is using the Stage Questionnaire (b) to assess a 24 - month-old child. What is the best intervention for the nurse to initiate after the assessment is completed? #Correct Answer- Provide the parents with a list of stimulating activities When caring for a child sickle cell disease, the nurse knows that the child will most likely exhibit which sign when experiencing a sickle cell crisis? #Correct Answer- Pain The nurse is administering an oral medication to a reluctant preschool-age boy. Which intervention should the nurse implement? #Correct Answer- Use straightforward approach with the child The nurse is planning care for a newborn infant scheduled for a cardiac catheterization. Which occurrence poses the greatest risk for this child? #Correct Answer- Acute hemorrhage from the entry site of the catheter after the procedure The school nurse is presenting a seminar to parents about child safety that focuses on prevention of spinal cord injuries. What information is most important for the nurse include in the teaching plan? #Correct Answer- Seat belt and car seat laws for use in motor vehicles should be reinforced A mother brings her 2-year-old son to the clinic because he has been crying and pulling on his earlobe for the past 12 hours. The child's oral temperature is 101.2 F (38 C). Which intervention should the nurse implement? #Correct Answer- Ask the mother if the child has had a runny nose A mother is concerned that her 3-year-old son wants to play with female doll figures. The child is not interested in building blocks, trucks, or other typical "boy" toys. How should the nurse respond to the mother's concern? #Correct Answer- Exploring different roles in imaginary play is typical at this age Which nursing intervention is most important to assist in detecting hypopituitarism and hyperpituitarism in children? #Correct Answer- Carefully recording the height and weight of children to detect inappropriate growth rates The parents of a 4 week-old infant phone the pediatric clinic to report that their infant eats well but vomits after each feeding. To differentiate between normal regurgitation and pyloric stenosis, which information is most important for the nurse to obtain? #Correct Answer- Degree of forcefulness of vomiting episodes A 4-month-old boy has an inguinal hernia that is visible when he cries, but it does not cause him discomfort. His parents ask if the hernia should be repaired now. The nurse's response should be based on what information? #Correct Answer- Surgical correction is indicated if the hernia is incarcerated A 2-year-old child with heart failure (HF) is admitted for replacement of a graft for coarctation of the aorta. Prior to administering the dose of digoxin (Lanoxin), the nurse obtains an apical heart rate of 128 beast| minute. What action should the nurse implement? #Correct Answer- Administer the scheduled dose The nurse is assisting the mother of child with phenylketonuria (PKU) to select foods that are in keeping with the child's dietary restrictions. Which foods are contraindicated for this child? #Correct Answer- Foods sweetened with aspartame During a routine physical exam, a male adolescent client tell the nurse, " Sometimes, my mother gets angry because I want to be with my own friends". What is the best initial response by the nurse? #Correct Answer- Ask about client's response to his mother's age Which response demonstrates that the mother of a young girl with a urinary tract infection (UTI) understands home care for the child? #Correct Answer- I will bring her back to the doctor's office for another urine test The nurse is caring for a one-year-old boy who has type 1 diabetes mellitus (DM). His mother asks how will she recognize hypoglycemia in her infant who cannot tell her how he feels. Which information should the nurse provide? #Correct Answer- Hypoglycemia in infants causes changes in behavior and cold clammy skin An 8 year-old child is admitted to the Emergency Department because of lower right quadrant pain, nausea, and vomiting. Which assessment of the abdomen should the nurse conduct after all other assessments are complete? #Correct Answer- Palpation An infant is admitted for surgery who has a Wilms' tumor. What nursing intervention should the nurse implement during the preoperative period? #Correct Answer- Careful bathing and handling that avoids abdominal manipulation A female of child - bearing age receives a rubella vaccination. She has two children at home, ages 13 months and 3 years. Which instruction is most important for the nurse to provide to this client? #Correct Answer- Encourage the client to immunize the children The teacher notifies the school nurse that a child's nose is bleeding for no apparent reason. What action should the nurse implement first? #Correct Answer- Pinch the nose using thumb and finger for 10 minutes A breastfeeding infant, screened for congenital hypothyroidism, is found to have low levels of thyroxine (T4) and high levels of thyroid stimulating hormone (TSH). What is the best explanation for this finding? #Correct Answer- The TSH is high because of the low production of T4 by the thyroid When administering indomethacin (Indocin) to a premature infant who has patent ductus arteriosus, the nurse should anticipate which outcome? #Correct Answer- Decreased cardiac murmur A hospitalized child stiffens and stars to seize as the nurse enters the room. What actions should the nurse take? (Select all apply) #Correct Answer- • Turn client to the side if possible • Pad side rails with available pillows and blankets • Monitor duration and progress of the seizure A middle school male student was recently diagnosed with Attention-Deficit Hyperactivity Disorder (ADHD) and is having trouble with his grades. He is referred to the school nurse by the teacher because he continues to have learning problems. Which action should the school nurse take? #Correct Answer- Refer the child to the school counselor for educational testing An adolescent boy is hospitalized with full-thickness (third degreed) burns to both hands following a house fire. Three days after his admission to the burned unit, the nurse notes that teenager's hands are becoming more edematous. Which intervention is most important for the nurse to include in this client's plan care? #Correct Answer- Assess radial pulses every 2 hours A 10-year-old boy has been seen frequently by the nurse over the past three weeks after school begins in the fall. He reports headaches, stomach, and difficulty sleeping. What intervention should the nurse implement? #Correct Answer- Ask the boy to describe a typical day at school A 6-year-old child is diagnosed with rheumatic fever and demonstrates associated chorea (sudden aimless movements of the arms and legs). Which information should the nurse provide to the parents? #Correct Answer- The chorea or movements are temporary and will eventually disappear A 3-year -old girl who has been blind since birth is hospitalized because of a compound fracture of the femur and is now in traction. Which intervention is best for the nurse to implement to address this child's blindness? #Correct Answer- Request parents bring familiar objects as a stuffed animal from home The nurse has provided discharge teaching to the mother of a premature infant. Which statement by the mother would indicated that the understands the importance of making sure that her baby gets the monthly palivizumab (Synagis) injection? #Correct Answer- The medication will protect my baby from respiratory syncytial virus A 16 -year-old adolescent with acute myelocytic leukemia is receiving chemotherapy (CT) via an implanted medication port at the out-patient oncology clinic. What action should the nurse implement when the infusion is completed? #Correct Answer- Flush the mediport with saline and a heparin solution The nurse is evaluating the effects of thyroid therapy used to treat a 5-month-old with hypothyroidism. Which behavior indicates that the treatment has been effective? #Correct Answer- Laughs readily, turns from back to side A pre-school-age girl who has been taking oral ampicillin (Omnipen, Polycillin) for 3 days for a non-respiratory condition repeatedly tells the nurse that her throat hurts. The child has no evidence of dyspnea or urticaria. Which action should the nurse implement? #Correct Answer- Explain gently that medication must be taken A 9-year-old boy is diagnosed with diabetes mellitus Type 1. Which stage of Erikson's theory of psychosocial development is the nurse addressing when teaching this client about insulin injections? #Correct Answer- Industry After receiving a single fluid bolus of 20 mg / kg of normal saline, a child's heart rate is 140 beats/ minute, blood pressure is 70/50, and capillary refill is 6 seconds. The child is anxious and crying. Which intervention should the nurse implement first? #Correct Answer- Repeat the normal saline bolus as prescribed The clinic nurse is assessing a 3-year -old child with sudden onset of irritability, thick muffled voice, and barking on inspiration. The child is febrile and leaning forward to breathe with tongue protruding, is drooling, and has suprasternal retractions. Which intervention should the nurse implement first? #Correct Answer- Alert the emergency response team The nurse is caring for an infant who was recently diagnosed with a congenital heart defect. Which assessment finding is most important for the nurse to report to the healthcare provider? #Correct Answer- Weight gain of 2.2 lbs (1kg) in las 48 hours The nurse administers digoxin (Lanoxin) to a 9-month-old infant with an apical heart rate of 160 beats per minute. Which apical pulse rate indicates that therapeutic effect of the medication has been achieved? #Correct Answer- 120 beats per minute Which toy is most appropriate for a 10 -year-old child with acute rheumatic fever who is on strict bed rest? #Correct Answer- Checkers A 12-year-old obese male comes to the clinic with his mother and a note from the school nurse for follow-up of Acanthosis Nigricans, a thickening and darkening of the skin. The child is concerned and anxious that he has a serious condition. How should the nurse respond? #Correct Answer- Ask the child and his mother what he was told about this condition A 10-year-old girl is diagnosed with inflammatory bowel disease (IBD). Her mother is concerned that she will experience developmental delays as the result of this disorder. How should the nurse respond? #Correct Answer- Growth failure is a concern, but developmental delays are not likely to occur Which instruction should the nurse include in the discharge teaching plan of a 7-year-old girl with a history of frequent urinary tract infections? #Correct Answer- Monitor for changes in urinary odor During her sports physical examination, 15-year -old female requests oral contraceptives. She explains that she is sexually active and does not want her parents to know. What action should the nurse take? #Correct Answer- Counsel the client about the risks and benefits of using oral contraceptives A child with acute laryngotracheobronchitis (croup) received epinephrine 2 hours ago in the emergency room, and is now being prepared for discharge to the home. The nurse should instruct the parents to take which action if the child's uncontrolled coughing reoccurs? #Correct Answer- Sit with the child in the bathroom with hot steam During a well baby clinic visit, the mother of a 6-month-old infant asks the nurse if she can have a prescription for Poly Vi Sol with fluoride. Though the infant is still breast feeding, the mother provides the child with supplemental formula feedings. Which assessment is most important for the nurse to obtain? #Correct Answer- Water source used with supplement feedings The mother of an 11 -year-old boy who has juvenile arthritis tells the nurse, "I really don't want my son to become dependent on pain medication, so I only allow him to take it when he is really hurting". Which information is most important for the nurse to provide this mother? #Correct Answer- Giving pain medication around the clock helps control the pain An 8-year-old male client with nephrotic syndrome is receiving salt-poor human albumin intravenously. Which finding indicate to the nurse that the child is manifesting a therapeutic response? #Correct Answer- Decreased periorbital edema A 12-year-old boy with leukemia is being discharged from the hospital with a white blood cell (WBC) count of 4,000/mm. He is scheduled to receive antineoplastic chemotherapy as an outpatient. What instruction should the nurse include in this child's discharge plan? #Correct Answer- Avoid eating at buffets, smorgasbords, and salad bars The nurse is assessing an 8-month-old who has a cough, axillary temperature of 100 F, and rhinorrhea. What information is most important for the nurse to obtain from this child's mother? #Correct Answer- Immunization status of the infant Which client requires immediate intervention by the nurse? #Correct Answer- A child with acute renal failure and hyperkalemia A 7-year-old male is referred to the school clinic because he fainted on the playground. His height is 3 feet, 7 inches (107.5 cm), he weighs 55 pounds ( 25 kg), and his body mass index (BMI) IS 20.9. Which assessment finding is most important for the nurse to address? #Correct Answer- Reports drinking 3 to 4 high calorie, carbonated beverages day The mother of 6-year-old girl is concerned about her child's obesity. The child's weight plots at the 75 percentile, and height at the 25 percentile. The child's body mass index (BMI) is at the 85 percentile for age and gender. Which interventions should the nurse implement? (Select all apply) #Correct Answer- • Determine the child's usual physical activity pattern • Obtain the child's 3 - day diet history based on the mother's input • Inquire as to whether or not the school has a physical education The nurse is examining an infant for possible cryptorchidism. Which exam technique should be used? #Correct Answer- Place the infant in warm room and use a calm approach An infant who has been diagnosed with a tracheoesophageal fistula (TEF). Which nursing intervention is indicated for this infant prior to surgical repair? #Correct Answer- Maintain nothing by mouth status An adolescent with non-Hodgkin's lymphoma (NHL) is complaining of sore mouth two days after begging chemotherapy. What activity should the nurse implement? #Correct Answer- Frequent use of saline oral rinses and soft sponge toothbrush The nurse is performing a routine examination of a 6-month-old infant at community health clinic. Records indicate that the child weighed 3 kg at birth. The clinic uses lbs to describe weight. When assessing this child, approximately what weight, in lbs, should the nurse consider to be within normal range for this child? #Correct Answer- 12 to 15 lb When development a teaching plan for an adolescent male who was recently diagnosed with Type 1 diabetes mellitus, the nurse should instruct the to eat a source of sugar if which symptom occurs? #Correct Answer- Profuse perspiration A breast feeding mother returns to work when her infant is 5 months old. She is having difficulty pumping enough milk to meet her infant's dietary requirements. Which suggestion should the nurse provide to this mother? #Correct Answer- Offer a follow-up transitional formula The nurse is assessing an infant with aortic stenosis and identifies bilateral fine crackles in both lung fields. Which additional finding should the nurse expect to obtain? #Correct Answer- Hypotension and tachycardia A child with possible Duchenne muscular dystrophy ( MD) undergoes an electromyogram (EMG). Following the procedure, the child's parents tell the nurse that the child is complaining of sore muscle. How should the nurse respond? #Correct Answer- Offer reassurance that muscle soreness following this procedures is temporary and does not indicate a problem The heart rate for a 3-year-old with a congenital heart defect has steadily decreased over the last few hours, and is now at 76 beats/minute; the previous reading 4 hours ago was 110 beats/minute. Which additional clinical finding should be reported immediately to the healthcare provider? #Correct Answer- Blood pressure of 70/40 The health care provider prescribe epinephrine 0.01 mg/kg IM for a child with asthma who weighs 55 pounds. The available medication is labeled, 1 mg/ml. Based on the child's weight, how many ml should the nurse administer? (Enter numerical value only. If rounding, round to the nearest hundredth) #Correct Answer- 0.25 ml After receiving a single fluid bolus of 20 mg/ kg of normal saline, a child's heart rate is 140 beats/minute, blood pressure is 70/50, and refill is 6 seconds. The child is anxious and crying. Which intervention should the nurse implement first? #Correct Answer- Repeat the normal saline bolus as prescribed The nurse should instruct the parent of an 8-year-old child who has sickle anemia to alert for which complaint from the child? #Correct Answer- "I'm really hot and thirsty" During a routine clinic visit, the nurse determines that 5-year-old girl's systolic blood pressure is greater than the 90th percentile. What action should the nurse implement next? #Correct Answer- Take the blood pressure two more time during the visit and determine the average of the three readings A child with hemophilia arrives at the clinic with a swollen knee after falling off a bicycle. What action should the nurse implement first? #Correct Answer- Apply ice pack and compression dressing to knee What snack is best to provide a 6-year-old child on prescribed while receiving treatment for osteomyelitis? #Correct Answer- Milk shake A one - month old male infant is brought to the clinic by his mother who states that her son has been vomiting forcefully after each meal for the last three days. The infant is afebrile, dehydrated, and pyloric stenosis is suspected. What other finding should the nurse identify that are consistent with pyloric stenosis? #Correct Answer- An olive-shaped mass in the abdominal area The nurse is evaluating a young child with atopic dermatitis. Which question should the nurse ask the parent while obtaining the child's history? #Correct Answer- Has the child displayed any symptoms of asthma or hay fever? A female infant recently admitted with vomiting and diarrhea now weighs 10 kg. Her weigh at a previous well-baby visit was 11 kg. What is the percentage of body weight loss for this infant? #Correct Answer- 9% A 3-month-old with myelomeningocele and atonic bladder is catheterized every 4 hours to prevent urinary retention. The home health nurse note that the child has developed episodes of sneezing, urticaria, watery eyes, and a rash in the diaper area. What action is most important for the nurse to take? #Correct Answer- Change to latex - free gloves when handling infant A 17-year-old male student with cystic fibrosis talks with the school nurse about his disease and wonders how it will affect getting married and having children. Which relevant information would the nurse include in this discussion? #Correct Answer- He is likely to have infertility problems and further evaluation A child weighing 67 pounds receives a prescription for benztropine (Cogentin) 0.61 mg IV q 12 hours. This drug is available as 1 mg/ml ampoules. How many ml should the nurse administer? (Enter the numeric value only. If rounding is required, round to the nearest hundredth) #Correct Answer- 0.61 ml/ dose A 12-year-old is admitted to the hospital with possible encephalitis, and a lumbar puncture is schedule. Which information should the nurse provide concerning to this procedure? #Correct Answer- Describe the side-lying, knee to the chest position that must be assumed during the procedure How should the nurse respond to the concerned parent of a 15-month-old who is not yet able to self-feed with a spoon? #Correct Answer- Suggest using foods that can be eaten with fingers A child who has been vomiting for 3 days is admitted for correction of fluid and electrolyte imbalances. What acid base imbalance is this child likely to exhibit? #Correct Answer- Metabolic alkalosis When providing care for a child who is in balanced suspension skeletal traction using a Thomas splint and Pearson attachment to the right femur, which intervention is most important for the nurse to implement? #Correct Answer- Monitor peripheral pulse and sensation in the leg The healthcare provider prescribes antipyrine and benzocaine (Auralgan Otic), and anesthetic ear drop, for a two-year-old child with otitis media in the right ear. After positioning the child with the affect ear up, what action should the nurse take? #Correct Answer- Pull pinna of the ear down and back The nurse is caring for a female client with scoliosis who had a posterior spinal fusion and is in a body jacked cast. Which assessment finding indicates to the nurse the client is developing cast syndrome? #Correct Answer- Abdominal distention. The healthcare provider prescribes amoxicillin (Amoxil) 80mg PO every 8 hours for a child who weighs 25 pounds. The suspension is labeled Amoxil 125mg/5 ml. How many ml should the child receive in a 24-hour period? (Enter numeric value only. If rounding is required, round to the nearest tenth. #Correct Answer- 9.6mL A child with Grave's Disease who is taking propranolol (Inderal) is seen in the clinic. The nurse should monitor the child for which therapeutic response? #Correct Answer- Decreased heart rate. A male infant is admitted to the pediatric unit with pertussis and is exhibiting a "whooping-like cough." The mother brings the infant to the nurse's station to seek assistance. Which intervention should the nurse implement first? #Correct Answer- Cover the infant's mouth and assist the mother to take the infant back to the room. The nurse is assessing a 3-year-old boy who attends a daycare center. Following an upper respiratory tract infection, he developed acute otitis media. Which factor places this child at greatest risk for developing acute otitis media? #Correct Answer- A child's Eustachian tube is shorter and straighter than an adult's Eustachian tube. When administering indomethacin (Indocin) to a premature infant who has patent ductus arteriosus, the nurse should anticipate which outcome? #Correct Answer- Decreased cardiac murmur. The nurse in the Emergency Center is triaging an 8-year-old boy who fell from a tree. The child is crying and complaining of pain in the left forearm. Which intervention should the nurse implement first? #Correct Answer- Apply a cold pack to his left forearm. Several children at a day camp return from playing in a tick-infested field. What action should the camp nurse take first? #Correct Answer- Observe the children's skin for attached ticks. A 2-year-old girl is brought to the clinic by her 17-year-old mother. When the nurse observes that the child is drinking sweetened soda from her bottle, what information should the nurse discuss with this mother? (Select all that apply.) #Correct Answer- • Drinking soda is related to childhood obesity. • Toddlers should be drinking from a cup by age 2 .• Dental caries are associated with drinking soda. The nurse is assessing a 2-year-old child. What behavior indicates that the child's language development is within normal limits? #Correct Answer- Half of child's speech is understandable A 2-year-old female infant is hospitalized for the surgical repair of an umbilical hernia. After returning to the postoperative neonatal unit, her respiratory rate and heart rate have increased during the last hour. Which intervention should the nurse implement? #Correct Answer- Administer a PRN analgesic prescription. The nurse is planning care for a 5 - month-old with gastroesophageal reflux disease whose weight has decreased by 3 ounces since the last clinic visit one month ago. To increase caloric intake and decrease vomiting, what instruction should the nurse provide this mother? #Correct Answer- Thicken formula with cereal for each feeding A 4-years-old boy was recently diagnosed with Duchenne muscular dystrophy (DMD). Which characteristic of the disease is most important for the nurse to focus on during the initial teaching? #Correct Answer- Lower legs become progressively weaker, causing a wedding, unsteady gait In caring for an client with acute epiglottitis, which nursing action takes priority? #Correct Answer- Prepare for endotracheal intubation Which client requires immediate intervention by the nurse? #Correct Answer- A Child with acute renal failure and hyperkalemia A child who is preparing to enter the first grade has recurring atopic dermatitis (eczema) and is brought to the clinic because of a recent exacerbation. Which suggestion should the nurse provide this child's parent? #Correct Answer- Ensure that the child's lunch at school is a hypoallergic diet. The mother of a toddler reports to the nurse working in the pediatric clinic that her child has had a fever and sore throat for the past two days. The nurse observes several swollen red spots in the child's body, a few of which are fluid filled blisters. What action should the nurse implement? #Correct Answer- Implement transmission precautions. A male toddler is brought to the emergency center approximately three hours after swallowing tablets from his grandmother's bottle of digoxin (Lanoxin). What prescription should the nursed implement first? #Correct Answer- Give IV digoxin immune fab (Digibind) When assessing a newborn, the nurse includes assessment for early signs of congenital hip dysplasia. Which finding is an indication of this condition? #Correct Answer- Asymmetry of the gluteal folds. An adolescent with pelvic inflammatory disease (PID) is admitted to the hospital after 14 days of taking levofloxacin (Levaquin) 500 mg orally once daily and metronidazole (Flagyl) 500 mg twice daily. She asks the nurse, "Why do I have to be in the hospital? Why can't I get my treatment at home?" Which purpose should the nurse provide that supports and effective outcome? #Correct Answer- Administration of a supervised parenteral antibiotic protocol. During the admission procedure of a 6-year-old, the child states, "I'm going to have an operation." Which response is best for the nurse to provide to this child? #Correct Answer- "Tell me what an operation is. A male infant with bronchiolitis is brought to the clinic by his mother. The infant is congested and febrile with a capillary refill time of 2 seconds. What information should the nurse discuss with the mother? #Correct Answer- Keep infant isolated from others. A 6-year-old boy with bronchial asthma takes the beta-adrenergic agonist agent albuterol (Proventil). The child's mother tells the nurse that she uses this medication to open her son's airway when he is having trouble breathing. What is the nurse's best response? #Correct Answer- Assure the mother that she is using the medication correctly. In developing a plan of care for a child with bacterial meningitis, which intervention should the nurse plan to implement? #Correct Answer- Maintain strict insolation after identification of the causative agent. The nurse determines that an infant admitted for surgical repair of an inguinal hernia voids a urinary stream from the ventral surface of the penis. What action should the nurse take? #Correct Answer- Document the finding. A mother brings her 3-month-old infant to the clinic because the baby does not sleep through the night. Which finding is most significant in planning care for this family? #Correct Answer- The diapers area shows severe skin breakdown. Insulin therapy is initiated for a 12-year-old child who is admitted with diabetic ketoacidosis (DKA). Which action is most important for the nurse to include in the child's plan of care? #Correct Answer- Monitor serum glucose for adjustment in infusion rate of Regular insulin (Novolin R). The mother of a 2-year-old boy consults the nurse about her son's increased temper tantrums. The mother states, "Yesterday he threw a fit in the grocery store, and I did not know what to do. I was so embarrassed. What can I do if this occurs again?" Which recommendation is best for the nurse to provide this mother? #Correct Answer- Walk away from him and ignore the behavior. The heart rate for a 3-year-old with a congenital heart defect has steadily decreased over the last few hours, and is now at 76 beats/minute; the previous reading 4 hours ago was 110 beats/minute. Which additional clinical finding should be reported immediately to the healthcare provider? #Correct Answer- Blood pressure of 70/40. A mother brings her 2-month-old son to the clinic for a well-baby exam. During the assessment the nurse finds that the right testicle is not descended into the scrotum but the left is palpable. Which action should the nurse take? #Correct Answer- Ask if the right testis has been seen in the scrotum before. A 6-month-old, diagnosed with short bowel syndrome, began enteral feedings yesterday. To maintain normal growth and development of the child during this period, what action should the nurse include in the infant's plan of care? #Correct Answer- Give the infant a pacifier during feeding. Which nursing problem has the highest priority when providing preoperative care for an infant born with bladder atrophy? #Correct Answer- Risk for infection related to impaired skin integrity. The nurse is giving an intramuscular injection of an antibiotic to a 16-month-old toddler with pneumonia. The toddler does not have any known allergies and been walking without assistance for one month. Which technique should the nurse select for administration? #Correct Answer- Administer the injection into the middle of the lateral aspect of the thigh. A clinic nurse is assessing infants and toddlers for fine and gross motor development. Which child should the nurse refer to a healthcare provider for further evaluation? #Correct Answer- 3-year-old preferring to walk on the tip toes. The nurse is assessing a 9-year-old boy who has been admitted to the hospital with possible acute post-streptococcal glomerulonephritis (APSGN). In obtaining his history, what information is most significant? #Correct Answer- A sore throat last week. An infant is born with a ventricular septal defect (VSD) and surgery is planned to correct the defect. The nurse recognizes that surgical correction is designed to achieve which outcome? #Correct Answer- Prevent the return of oxygenated blood the lungs. In developing a behavior modification program for an extremely aggressive 10-year-old boy, what should the nurse do first? #Correct Answer- Determine what activities, foods, and toys the child enjoys. An adolescent who is taking antiretroviral therapy for HIV infection arrives at the clinic for a follow up visit. Which information is most important for the nurse to obtain? #Correct Answer- Missed medication doses The nurse is caring for a 3-year old child who is 2 hours postop from a cardiac catheterization via the right femoral artery. Which assessment finding is an indication of arterial obstruction? #Correct Answer- Right foot is cool to the touch and appears pale and blanched. Following a motor vehicle collision, a 3-year old girl has a Spica cast applied. Which toy is best for the nurse for this 3-year-old child? #Correct Answer- Set of cloth and hand puppets An infant with tetralogy of Fallot becomes acutely cyanotic and hyper apneic. Which action should the nurse implement first? #Correct Answer- Place the infant in a knee-chest position A child admitted with diabetic ketoacidosis is demonstrating Kussmaul's respirations. The nurse determines that the increased respiratory rate is a compensatory mechanism for which acid base alteration? #Correct Answer- Metabolic acidosis 7 years old is admitted to the hospital with persistent vomiting, and a nasogastric tube attached to low intermittent suction is applied. Which finding is most important for the nurse to report to the healthcare provider? #Correct Answer- Serum potassium of 3.0 mg/dL. The nurse is evaluating diet teaching for a client who has nontropical sprue (celiac disease). Choosing which food indicates that the teaching has been effective? #Correct Answer- Creamed corn During a well-baby check, the nurse hides a block under the baby's blanket, and the baby looks for the block. Which normal growth and development milestone is the baby developing? #Correct Answer- Object permanence The nurse is measuring the frontal occipital circumference (FOC) of a 3-months old infant, and notes that the FOC has increased 5 inches since birth and the child's head appears large in relation to body size. Which action is most important for the nurse to take next? #Correct Answer- Palpate the anterior fontanel for tension and bulging The nurse is preparing a 10-year-old with a lacerated forehead for suturing. Both parents and 12-year-old sibling are at the child's bedside. Which instruction best supports family? #Correct Answer- Please decide who will stay when the healthcare provider begins suturing While teaching a parenting class to new parents the nurse describes the needs of infants and toddlers regarding discipline and limit setting. What is the most important reason for implementing such parenting behaviors? #Correct Answer- They provide the child with a sense of security The parents of a newborn infant with hypospadias are concerned about when the surgical correction should occur. What information should the nurse provide? #Correct Answer- Repairs typically should be done before the child is potty trained. Which drink choice on a hot day indicates to the nurse that a teenager with sickle cell anemia understands dietary consideration related to the disease? #Correct Answer- Lemonade. The nurse is assessing an infant with diarrhea and lethargy. Which finding should the nurse identify that is consistent with early dehydration? #Correct Answer- Tachycardia While auscultating the lung sounds of a 5-year-old Chinese boy who recently completed antibiotic therapy for pneumonia, the nurse notices symmetrical, round, bruise-like blemishes on his chest. What action is best for the nurse to take? #Correct Answer- Inquire about the use of alternative methods of treatment A child with acute lymphocytic leukemia (ALL) who is receiving chemotherapy via a subclavian IV infusion, has an oral temperature of 103 degrees. In assessing the IV site, the nurse determines that there are no signs of infection at the site. Which intervention is the most important for the nurse to implement? #Correct Answer- Obtain specimen for blood cultures A child who weighs 25 kg is receiving IV ampicillin 300 mg/kg/24 hours in equally divided doses every 4 hours. How many mgs should the nurse administer to the child for each dose? #Correct Answer- 1875mg The nurse is caring for an infant scheduled for reduction of intussusceptions. The day before the scheduled procedure the infant passes a soft-formed brown stool. Which intervention should the nurse implement? #Correct Answer- Notify the healthcare provider of the passage of brown stool. The mother of a 4-month old asks the nurse for advice in preventing diaper rash. What suggestion should the nurse provide? #Correct Answer- Use a barrier cream, such as zinc oxide, which does not have to be completely removed with each diaper change. Which statement by a school aged client going to summer camp indicates the best understanding of the mode of transmission of Lyme disease? #Correct Answer- I have to wear long sleeves and pants when we're hiking around the pond. The HR for a 3-year-old with a congenital heart defect has steadily decreased over the last few hours, now it's 76 bpm, the previous reading 4 hours ago was 110 bpm. Which additional finding should be reported immediately to a healthcare provider? #Correct Answer- BP 70/40. the parents of a 3 y/o boy who has Duchenne muscular dystrophy (DMD) ask "how can our son have this disease? We are wondering if we should have any more children" What information should the nurse provide these parents? #Correct Answer- this is an inherited X-linked recessive disorder, which primarily affects male children in the family The nurse finds a 6 month old infant unresponsive and calls for help. After opening the airway and finding that the infant is still no breathing. Which action should the nurse take? #Correct Answer- Give two breaths that makes the chest rise A 3-year-old with HIV infection is staying with a foster family who is caring for 3 other foster children in their home. When one of the children acquires pertussis, the foster mother calls the clinic and asks the nurse what she should do. Which action should the nurse take first? #Correct Answer- Review the immunization documentation of the child who has HIV A 16 y/o female student with a history of asthma controlled with both an oral antihistamine and an albuterol (Proventil) metered-dose inhaler (MDI) comes to the school nurse. The student complains that she cannot sleep at night, feels shaky and her heart feels like it is "beating a mile per minute" Which information is most important for the nurse to obtain? #Correct Answer- How often the MDI is used daily The nurse is assessing a child for neurological soft signs, which finding is most likely demonstrated in the child's behavior? #Correct Answer- Poor coordination and sense of position The nurse is assessing an infant with pyloric stenosis. Which pathophysiological mechanism is the most likely consequence of this infant's clinical picture? #Correct Answer- Metabolic alkalosis A 4 month-old girl is brought to the clinic by her mother because she has had a cold for 2 o 3 days and woke up this morning with a hacking cough and difficulty breathing. Which additional assessment finding should alert the nurse that the child is in acute respiratory distress? #Correct Answer- flaring of the nares a two-year-old boy begins to cry when the mother starts to leave. What is the nurse's best response in this situation? #Correct Answer- Let me read this book to you A child with leukemia is admitted for Chemotherapy and the nursing diagnosis "altered nutrition, less those body requirements related to anorexia, nausea and vomiting" is identified. Which intervention the nurse included in this child plan of care? #Correct Answer- Allow the child to eat any food desired and tolerated a 6-year-old who has asthma is demonstrating a prolonged expiratory phase and wheezing, and has a35% of personal best peak expiratory flow rate (PEFR) based on these finding, actions should the nurse take first? #Correct Answer- Administer a prescribed bronchodilator The nurse plans to administer 10 mcg/kg of digoxin elixir as a loading dose to a child who weighs 55 pounds. Digoxin is available as elixir of 50 mcg/ml. How many ml of the digoxin elixir should the nurse administer to this child? #Correct Answer- 5ml the nurse observes a mother giving her 11 month-old ferrous sulfate, followed by two ounces of orange juice. What should the nurse do next? #Correct Answer- Give positive feedback about the way she administered the sulfate Which nursing intervention is most important to include in the plan of care for a child with acute glomerulonephritis #Correct Answer- weight the child daily During a well-baby visit the parents explain that a soft bulge appears in the groin of their 4-month old son when he cries or strain stooling. The infant is schedule for surgical repair of the inguinal; hernia in two weeks. The parent should be instructed to take which measure if the hernia becomes incarcerated prior to the surgery? #Correct Answer- Gently manipulate the hernia for reduction A 16-year-old male client who has been treated in the past for a seizure disorder is admitted to the hospital. Immediately after admission he begins to have a grand mal seizure. Which action should the nurse take? #Correct Answer- Observe him carefully The mother of a 9-month old who was diagnosed with respiratory syncytial virus yesterday calls the clinic to inquire if it will be all right to take her infant to the first b-day party of a friend's child the following day. What response should the nurse provide this mother? #Correct Answer- Do not expose other children to RSV. It is very contagious even without direct contact When screening a 5-year-old for strabismus, what action should the nurse take #Correct Answer- Direct the child through the six cardinal position of glaze The nurse is assessing a 6-month old infant. Which response requires further evaluation by the nurse? #Correct Answer- Demonstrate startle reflex A child is brought to the clinic complaining of fever and joins pain, and is DX with rheumatic fever. When planning care for this child what is the goal of nursing care? #Correct Answer- Prevent cardiac damage The nurse working on the pediatric unit takes two 8-year old girls to the playroom. Which activity is best for the nurse to plan for these girls? #Correct Answer- Selecting a board game The nurse is developing the plan of care for a hospitalized child with Von Willebrand disease. What priority nursing intervention should be included in this child plan of care #Correct Answer- Guard against bleeding injuries How should the nurse instruct the parents of a 4 month old with seborrheic dermatitis (cradle cap) to shampoo the child's hair? #Correct Answer- Use a soft brush and gently scrub the area Prior to discharge, the parents of a child with cystic fibrosis are demonstrating chest physiotherapy (CPT) that they will perform for their child at home. Which action requires intervention by the nurse? #Correct Answer- The child is placed in a supine position to begin percussion When assessing the breath sounds of an 18-month old child who is crying, what action should the nurse take? #Correct Answer- Allow the child to initially play with stethoscope, and distract during auscultation The mother of a one-month old calls the clinic to report that the back of her infant is flat. How should the nurse respond? #Correct Answer- Position the infant on the stomach occasionally when awake and active A 7-year-old child is admitted to the hospital with acute glomerulonephritis (AGN). When obtaining the nursing history which finding should the nurse expect to obtain? #Correct Answer- A recent strep throat infection The nurse plans to screen only the highest risk children for scoliosis. Which group of children should the nurse screen first #Correct Answer- Girls between ages 10 and 14 In assessing a 10-year-old newly diagnosed with osteomyelitis, which information is most for the nurse to obtain. #Correct Answer- Recent recurrence of infections A 3-year-old boy in a daycare facility scratches his head frequently and the nurse confirms the presence if head lice. The nurse washes the child's hair with permethrin(Nix) shampoo and call his parents. What instructions should the nurse provide to the parents about treatment of head lice? #Correct Answer- Wash the child's bed linens and clothing in hot soapy water The nurse on a pediatric unit observes a distraught mother in the hallwayher 3year old son for wetting his pants. What initial action should the nurse take? #Correct Answer- Inform the mother that toilet training is slower for boys During a well child visit for their child, one of the parents who have an autosomal dominant disorder tells the RN, "We don't plan on having any more children, the next child is likely to inherit this disorder." How should the RN respond? #Correct Answer- Confirm that there is a 50% chance of their future child inheriting this disorder. The RN is palpating the lymph nodes of an 18 month old. Which finding should the RN call to the attention of the healthcare provider? #Correct Answer- Enlarged, warm, tender, pre-auricular node. A 3-year-old boy was successfully toilet trained prior to his admission to the hospital for injuries sustained from a fall. His parents are very concerned about this regression in toileting. Which information should the RN provide to the parents? #Correct Answer- Children usually resume their toileting behaviors when they leave the hospital. A mother brings her 8 mo. old baby boy to clinic because he has been vomiting and had diarrhea for last 3 days. Which assessment is most important for nurse to make? #Correct Answer- Measure the infant's pulse While obtaining the vital signs of a 10-year-old who had a tonsillectomy this morning, the nurse observes the child swallowing every 2-3 minutes. Which assessment should the nurse implement? #Correct Answer- Inspect the posterior oropharynx A mother brings her 3-week old infant to the clinic because the baby vomits after eating and always seems hungry. Further assessment indicates that the infant's vomiting is projectile, and the child seems listless. Which additional assessment finding indicates the possibility of a life threatening complication? #Correct Answer- Irregular palpable pulse The nurse is performing a routine assessment of a 3-year old at a community health center. Which behavior by the child should alert the nurse to request a follow-up for a possible autistic spectrum disorder? #Correct Answer- Performs odd repetitive behaviors Following admission for cardiac catheterization, the nurse is providing discharge teaching to the parents of a 2-year-old toddler with Tetralogy of Fallot. What instruction should the nurse give the parents if their child becomes pale, cool, lethargic? #Correct Answer- Contact their HCP immediately During a follow up clinical visit a mother tells the nurse that her 5-month old son who had surgical correction for tetralogy of fallot has rapid breathing, often takes a long time to eat, and requires frequent rest periods. The infant is not crying while being held and his growth is in the expected range. Which intervention should the nurse implement? #Correct Answer- Auscultate heart and lungs while infant is held The mother of a 4-month-old baby girl asks the nurse when she should introduce solid foods to her infant. The mother states, "My mother says I should put rice cereal in the baby's bottle now." The nurse should instruct the mother to introduce solid foods when her child exhibits which behavior? #Correct Answer- Opens mouth when food comes her way A mother brings her school-aged daughter to the pediatric clinic for evaluation of her anti-epileptic medication regimen. What information should the nurse provide to the mother? #Correct Answer- The medication dose will be tapered over a period of 2 weeks when being discontinued A child receives a prescription for amantadine 42 mg PO BID. Amantadine is available as a 50 mg/5 mL syrup. Using a supplied calibrated measuring device, how many mL should the nurse administer per dose? (round to nearest tenth) #Correct Answer- 4.2 ml A mother of a 3-year old boy has just given birth to a new baby girl. The little boy asks the nurse, "why is my baby sister eating my mommy's breast?" how should the nurse respond? Select all that apply #Correct Answer- a. Remind him that his mother breastfed him too c. Reassure the older brother that it does not hurt his mother d. Explain that newborns get milk from their mothers in this way Nurse giving IV antibiotics to a 16 month old with pneumonia. No allergies and been walking without assistance for 1 month. Which technique should select for administration? #Correct Answer- Admin in middle of lateral aspect of thigh Nurse examining an infant for possible cryptorchidism. Which exam technique to use? #Correct Answer- Place in warm room and use calm approach Infant with diagnosis of TEF. What intervention is indicated prior to surgical repair? #Correct Answer- Maintain NPO. Nurse gets a call from a mother of a 10 y/o that just returned from camp with expanding circular red rash on arm. Mom asks what over the counter product is safe to use. How should the nurse respond? #Correct Answer- Explain need for immediate exam Breast feeding mom returns to work when baby is 5 months old. She has difficulty pumping enough to meet requirements for baby. Which suggestions does the nurse give? #Correct Answer- Supplement iron rich formula Toddler with hemophilia being discharged. Which teaching to include in discharge instruction? #Correct Answer- Apply padding on the sharp edges of furniture Hcp prescribes antipyretic and benzocaine (auralgan otic) eardrops for a 2 y/o with otitis media in right ear. Positioning child with affected ear up, what action to take? #Correct Answer- Pull pinna down and back During a routine clinic visit, 5 y/o SBP >90th percentile. What action is implemented next? #Correct Answer- Measure two more times then take average Nurse caring for a 1 y/o with DM type 1. Mom asks how to know signs of hypoglycemia. Info to provide? #Correct Answer- Change in behavior and cold and clammy skin Female recently admitted with vomiting and diarrhea now weighs 10 kg. Previous visit was 11 kg. What is the percent of body weight loss? #Correct Answer- 9% Nurse eval child with atopic dermatitis. Which question should nurse ask while obtaining history? #Correct Answer- Has child displayed any symptoms of asthma or hay fever 1-month old vomiting forcefully times 3 days after meals. Afebrile, dehydrated and pyloric stenosis is suspected. What other findings will identify to confirm this? #Correct Answer- Olive shaped mass Hospitalized child stiffens and starts to seize as the nurse enters the room. What action should nurse take? #Correct Answer- Turn patient if possible, pad rails with available pillows and blankets, monitor time and progress 12 y/o boy with leukemia discharges with a WBC of 4000. He is scheduled to receive chemo as outpatient. What instruction should nurse include in discharge plan? #Correct Answer- Avoid buffet and salad bars Nurse is planning care for a newborn scheduled for cardiac catheter. Which occurrence poses greatest risk? #Correct Answer- Acute hemorrhage from access site after procedure When providing care for a kid in balanced suspension skeletal traction using them as splint and attach to right femur. Which intervention most important? #Correct Answer- Monitor pulses and sensation 10 y/o with IBS. Mom concerned she will experience developmental delays as a result. How should the nurse respond? #Correct Answer- Growth failure is a concern but not developmental delays Adolescent with AHL complaining of sores in mouth 2 days after start of chem. What activity should nurse implement? #Correct Answer- Frequent NS oral rinse and soft sponge toothbrush What snack is best to provide 6 y/o prescribed bed rest while receiving treatment for osteomyelitis? #Correct Answer- Milkshake During well visit, parent has autosomal dominant trait disorder tells nurse "we don't plan on having any more kids because next will be likely to have disorder." How should nurse respond? #Correct Answer- Confirm that there is a 50% chance next kid will have disorder Nurse eval effects of thyroid therapy used to treat 5 months old with hypothyroidism. Which behavior indicates effective? #Correct Answer- Laughs readily, turns from back to side Child weighing 67 lbs receives prescription of Cogentin 0.61 mg IV q 12 hrs. Available is 1 mg/ml. How many ml to admin? #Correct Answer- 0.61 ml The heart rate for a 3-year-old with a congenital heart defect has steadily decreased over the last few hours, and is now at 76 beats/minute, the previous reading 4 hours ago was 110 beats/minute. Which additional clinical finding should be reported immediately to the healthcare provider? #Correct Answer- Blood pressure of 70/40 A 10-year-old boy has been seen frequently by the school nurse over the past three weeks after school begins in the fall. He complains of headaches, stomach aches, and difficulty sleeping. What intervention should the nurse implement? #Correct Answer- Ask the boy to describe a typical day at school A child with laryngotracheobronchitis (croup) received epinephrine 2 hours ago in the emergency room, and is now being prepared for discharge to home. The nurse should instruct the parents to take whish action if the child's uncontrolled coughing reoccurs? #Correct Answer- Sit with the child in the bathroom with hot steam After receiving a single fluid bolus of 20 mL/kg of normal saline, a child's heart rate is 140 beats/minute, capillary refill is 6 seconds. The child is anxious and crying. Which intervention should the nurse implement? #Correct Answer- Repeat the normal saline bolus as prescribed A 3 year old girl who has been blind since birth is hospitalized because of as compound fracture of the femur and is now in traction. Which intervention is best for the nurse to implement to address this child's blindness? #Correct Answer- Request parents bring familiar objects such as a stuffed animal from home Child with Duchenne MD undergoes EMG. Following procedure, child's parents tell nurse that child is complaining of sore muscles. How should nurse respond? #Correct Answer- Offer reassurance that muscle soreness is normal 4-month old with inguinal hernia visible when crying. Does not hurt him. Parents ask is they should repair that now. Nurse's response based on what info? #Correct Answer- Surgical correction if hernia incarcerated Prescriber prescribes Midazolam 0.08 mg/kg IM times 1 dose during conscious sedation for 2 y/o weighing 29 lbs. how many mg per dose? #Correct Answer- 1mg Nurse instructs parents of 8 y/o with SCA to be alert for which complaint? #Correct Answer- "I'm really hot and thirsty" Mom of a 6 y/o concerned about child's obesity. Child weight plot in 75th percentile and height in 25th percentile. BMI at 85% for age and gender. What interventions to implement? #Correct Answer- Obtain child's 3-day diet history based on mom, determine child's usual pattern of activity, inquire if school has a PE program Nurse assessing infant with AS and identifies bilateral fine crackles in both lung fields. Which additional finding should nurse expect? #Correct Answer- Hypotension, tachycardia Performing a routine exam on 6-month old at clinic. Records indicate baby weighed 3 kg at birth. Clinic uses pounds to describe weight. When assessing this child, approximately what weight in lbs should nurse consider to be within normal range? #Correct Answer- 12-15 lbs 12 y/o admitted with poss encephalitis and a lumbar puncture is scheduled. What info to provide kid about procedure? #Correct Answer- Describe side lying position that must be assumed during procedure Child vomiting for 3 days is admitted for fluid and electrolyte imbalance. What acid base imbalance is likely? #Correct Answer- Metabolic alkalosis 3 month old with myelomeningocele and anatomic bladder is cath every 4 hours to prevent urinary retention. Home health nurse said child developed sneezing, watery eyes, urticarial, and diaper rash. What action is most important for nurse to take? #Correct Answer- Change to latex free gloves when handling the baby Teacher notifies school nurse that kid has nose bleed for no reason. What action should nurse implement first? #Correct Answer- Pinch nose and hold for 10 minutes Nurse provides discharge teaching to mom of preemie. Which statement indicates importance of monthly Synagis injection? #Correct Answer- Protect baby from getting RSV Which client requires immediate intervention by nurse? #Correct Answer- ARF with hyperkalemia Albumin 25% for a kid with nephrotic syndrome. Which assessment finding indicates medication is effective? #Correct Answer- Reduction of edema When developing a plan of care for adolescent with a recent diagnosis of DM type 1, nurse should instruct to eat a source of sugar if which symptom occurs? #Correct Answer- Profuse sweating ####################################CONTINUED.......................................................... [Show More]

Last updated: 1 year ago

Preview 1 out of 352 pages

Reviews( 0 )

Recommended For You

 *NURSING> EXAM REVIEW > NURS 6501 Midterm Exam Review Guide (Weeks 1-6. Compilation in 99 Pages) (All)

preview
NURS 6501 Midterm Exam Review Guide (Weeks 1-6. Compilation in 99 Pages)

Nurs 6501 Midterm Exam Review Guide (Weeks 1-6) Cellular Processes and the Genetic Environment 1. Describe cellular processes and alterations within cellular processes. 2. What is the impact of t...

By SuperSolutions© , Uploaded: Nov 24, 2020

$15

 *NURSING> EXAM REVIEW > NUR2063 / NUR 2063 Essentials of Pathophysiology Exam Review Latest Update Rasmussen College (All)

preview
NUR2063 / NUR 2063 Essentials of Pathophysiology Exam Review Latest Update Rasmussen College

NUR 2063 Essentials of Pathophysiology Exam Review 1. A potentially lethal condition in which there is an acute elevation of circulating thyroid hormones is called _______________ - ANS: Thy...

By quiz_bit , Uploaded: Oct 28, 2020

$12

 *NURSING> EXAM REVIEW > Multidimensional Care (MDC) 1 MDC EXAM 2 REVIEW -. (Rasmussen College) (All)

preview
Multidimensional Care (MDC) 1 MDC EXAM 2 REVIEW -. (Rasmussen College)

MDC EXAM 2 REVIEW 11:19:19 1. The effects of Immobility a) Interventions that improve flexibility • P.R.E.P.(Perform passive ROM, Reposition Q2HR, Encourage independent activity as much as possible...

By SuperSolutions© , Uploaded: Dec 11, 2020

$11

 *NURSING> EXAM REVIEW > NUR 2356 / NUR2356 Multidimensional Care I (MDC I) Exam 2 Review. Rasmusssen College (All)

preview
NUR 2356 / NUR2356 Multidimensional Care I (MDC I) Exam 2 Review. Rasmusssen College

1. What is a function of the musculoskeletal system? -Assist with movement 2. As a nurse you know that during aging, a normal musculoskeletal change would be? -A patient that came to the cli...

By nurse_steph , Uploaded: Nov 15, 2020

$11

 *NURSING> EXAM REVIEW > NR566 / NR 566 Advanced Pharmacology for Care of the Family Final Exam Week 5 Review (Graded A) Chamberlain College of Nursing (All)

preview
NR566 / NR 566 Advanced Pharmacology for Care of the Family Final Exam Week 5 Review (Graded A) Chamberlain College of Nursing

NR-566 Advanced Pharmacology for Care of the Family Final Exam Week 5 Review 1. What therapy and its dose is started within 48 hours of a TIA or ischemic stroke? - 2. Causes of sickling / S...

By quiz_bit , Uploaded: Oct 08, 2020

$9

 *NURSING> EXAM REVIEW > NUR 2063 / NUR2063 Essentials of Pathophysiology Final Exam Review Modules 1 & 2 & 3 Latest Update Rasmussen (All)

preview
NUR 2063 / NUR2063 Essentials of Pathophysiology Final Exam Review Modules 1 & 2 & 3 Latest Update Rasmussen

Modules 1 & 2 & 3 Essentials of Pathophysiology Final Exam Review

By quiz_bit , Uploaded: Nov 02, 2020

$15

 *NURSING> EXAM REVIEW > NURS 6501 / NURS6501 Advanced Pathophysiology - Module 8 Knowledge Check (Latest Update) Walden University (All)

preview
NURS 6501 / NURS6501 Advanced Pathophysiology - Module 8 Knowledge Check (Latest Update) Walden University

Review Test Submission: Module 8 Knowledge Check

By quiz_bit , Uploaded: Jan 13, 2021

$9.5

 *NURSING> EXAM REVIEW > NR566 / NR 566 Advanced Pharmacology for Care of the Family Week 3 Quiz Study Guide . (Graded A) Chamberlain College of Nursing (All)

preview
NR566 / NR 566 Advanced Pharmacology for Care of the Family Week 3 Quiz Study Guide . (Graded A) Chamberlain College of Nursing

NR-566 Advanced Pharmacology for Care of the Family Week 3 Quiz Study Guide If a patient is taking an ACEI and develops a persistent dry cough, what should the provider do? Which cl...

By quiz_bit , Uploaded: Oct 08, 2020

$15

 *NURSING> EXAM REVIEW > NR566 / NR 566 Advanced Pharmacology for Care of the Family Midterm Exam Review . (Graded A) Chamberlain College of Nursing (All)

preview
NR566 / NR 566 Advanced Pharmacology for Care of the Family Midterm Exam Review . (Graded A) Chamberlain College of Nursing

NR-566 Advanced Pharmacology for Care of the Family NR566 Midterm Differentiate between primary and secondary hyperthyroidism Primary is the term used when the pathology is within the thyro...

By quiz_bit , Uploaded: Nov 21, 2020

$14

 *NURSING> EXAM REVIEW > NURS6650 / NURS 6650 Midterm Exam Walden University Review Latest Update (All)

preview
NURS6650 / NURS 6650 Midterm Exam Walden University Review Latest Update

NURS 6650 - Psychotherapy with Groups and Families Midterm Exam Review 1. The PMHNP is providing family therapy to two parents and their twin 14-year-old children. The children become defian...

By quiz_bit , Uploaded: Nov 09, 2020

$9.5

$24.00

Add to cart

Instant download

Can't find what you want? Try our AI powered Search

OR

GET ASSIGNMENT HELP
143
0

Document information


Connected school, study & course



About the document


Uploaded On

Sep 28, 2022

Number of pages

352

Written in

Seller


seller-icon
Expert1

Member since 4 years

882 Documents Sold


Additional information

This document has been written for:

Uploaded

Sep 28, 2022

Downloads

 0

Views

 143

Document Keyword Tags

THE BEST STUDY GUIDES

Avoid resits and achieve higher grades with the best study guides, textbook notes, and class notes written by your fellow students

custom preview

Avoid examination resits

Your fellow students know the appropriate material to use to deliver high quality content. With this great service and assistance from fellow students, you can become well prepared and avoid having to resits exams.

custom preview

Get the best grades

Your fellow student knows the best materials to research on and use. This guarantee you the best grades in your examination. Your fellow students use high quality materials, textbooks and notes to ensure high quality

custom preview

Earn from your notes

Get paid by selling your notes and study materials to other students. Earn alot of cash and help other students in study by providing them with appropriate and high quality study materials.


$24.00

WHAT STUDENTS SAY ABOUT US


What is Browsegrades

In Browsegrades, a student can earn by offering help to other student. Students can help other students with materials by upploading their notes and earn money.

We are here to help

We're available through e-mail, Twitter, Facebook, and live chat.
 FAQ
 Questions? Leave a message!

Follow us on
 Twitter

Copyright © Browsegrades · High quality services·